Download as pdf or txt
Download as pdf or txt
You are on page 1of 100

myknowledgeplus.nejm.

org 0 ri\-1 Lr
NEJM Knowledge+ N Pediatrics Board Review

(REJM
Knowledge+ Pediatrics Board Review El
Sync
Th

What is the most appropriate next step for a 4-year-old boy who presents with
6 days of persistent fever (despite having a negative strep test and beginning
a course of amoxicillin), bilateral nonexudative conjunctivitis, a maculopapular
truncal rash, erythema of the lips and tongue, and edema and erythema of the
hands and feet?

0% Initiate intravenous oxacillin

Initiate treatment with intravenous immunoglobulin G and high-dose


91%
aspirin

6% Obtain an electrocardiogram

Order tests for Epstein-Barr virus immunoglobulin M and


1%
i mmunoglobulin G

1% Initiate treatment with intravenous glucocorticoids

KEY LEARNING POINT

A child who meets classic clinical criteria for


Kawasaki disease should be treated with
intravenous immunoglobulin G and high-dose
aspirin.
1u3
CHALLENGE US NEXT QUESTION

Dashboard My Reports My Schedule Help About Contact Us Search Learning terd— /logv area 9 Sign Out

NEJM Knowledge+ is a product of NEJM Group, a division of the Massachusetts Medical Society. Copyright ©2020 Massachusetts Medical Society. All rights reserved.
myknowledgeplus.nejm.org

NEJM Knowledge+ N Pediatrics Board Review

EJM
Knowledge+ Pediatrics Board Reviews
smc

YOUR ANSWER IS CORRECT 100%

A 2-month-old girl is brought to the emergency department with a 3-day Which one of the following diagnoses is most likely in this case?
history of increasing irritability and respiratory distress. Her mother
states that she becomes extremely irritable and diaphoretic upon
breastfeeding but that she has not had any fever, congestion, diarrhea,
OA Kawasaki disease
or known exposure to sick contacts. Her mother reports that she has a
fussy temperament and has had some irritability with feeds since a
14% Tachycardia-induced cardiomyopathy
couple weeks after birth.

The patient is afebrile and has a heart rate of 180 beats per minute, a 141, Viral myocarditis
blood pressure of 84/47 mm Hg, a respiratory rate of 67 breaths per
46% Ventricular septa! defect
minute, and an oxygen saturation of 93% while breathing ambient air.
She is extremely fussy on examination and slightly diaphoretic. Her
cardiac examination is notable for a 2/6 blowing holosystolic murmur
26% Anomalous origin of the left coronary artery from the pulmonary artery
heard loudest at the apex with radiation to the left axilla and an
gallop. She has intercostal and subcostal retractions with faint crackles
heard at both lung bases. Her liver edge is 3 cm below the right costal
KEY LEARNING POINT
margin. She is slightly cool to the touch on her distal extremities, with a
capillary refill of 3 to 4 seconds.
The most likely diagnosis in an afebrile infant
Blood cultures and a respiratory pathogen panel are pending. Serum presenting with heart failure and
creatinine and aminotransferase levels are within normal limits. The electrocardiographic signs of myocardial ischemia is
leukocyte count is 9000 per mm3 (reference range, 5000-19,500), the
C-reactive protein level is 18 mg/liter (0-5), and the erythrocyte
anomalous origin of the left coronary artery from the
pulmonary artery. 1ED
sedimentation rate is 15 mm/hr (0-10). The brain natriuretic peptide
level is >2500 pg/mL (<100). RESOURCES
A chest radiograph (figure 1) and electrocardiogram (figure 2) are
obtained.
CHALLENGE US NEXT QUESTION

Dashboard My Reports My Schedule Help About Contact Us Search Learning technology I area9 Sign Out

NEJM Knowledge+ is a product of NEJM Group, a division of the Massachusetts Medical Society. Copyright ©2020 Massachusetts Medical Society. All rights reserved.
myknowledgeplus.nejm.org 0 ri\-1 Lr

NEJM Knowledge+ 1,(-1\:f Pediatrics Board Review

(REJM
Knowledge+ Pediatrics Board Review El
Sync
M

YOUR ANSWER IS CORRECT II 1 100%

Which one of the following diagnoses is most likely in this case?

0% Kawasaki disease

14% Tachycardia-induced cardiomyopathy

14% Viral myocarditis

46% Ventricular septa! defect

26% Anomalous origin of the left coronary artery from the pulmonary artery

KEY LEARNING POINT i mL


The most likely diagnosis in an afebrile infant
presenting with heart failure and
electrocardiographic signs of myocardial ischemia is
anomalous origin of the left coronary artery from the
pulmonary artery.

RESOURCES

CHALLENGE US NEXT QUESTION


EJNI
Kno%vledge .
Ci===1.11

Dashboard My Reports My Schedule Help About Contact Us Search Learning technology k area
=EN
NEJM Knowledge+ is a product of NEJM Group, a division of the Massachusetts Medical Society. Copyright ©2020 Massachusetts Medical Society. All rights reserved.
myknowledgeplus.nejm.org 0 1-1) Lr
NEJM Knowledge+ N Pediatrics Board Review

(REJM
Knowledge+ Pediatrics Board Review El
Sync
M

YOUR ANSWER IS CORRECT

Which one of the following antibiotic regimens is the most appropriate empiric
therapy for an 8-year-old boy with a prosthetic aortic valve placed 6 years ago
who presents with high fever; night sweats; fatigue; diminished appetite; small
erythematous, nontender, nonblanching lesions on the hands; and a new
partial dehiscence of the aortic valve on echocardiogram?

15% Ceftriaxone

1% Ciprofloxacin

18% Piperacillin-tazobactam

63% Ampicillin-sulbactam and gentamicin

2% Levofloxacin

KEY LEARNING POINT

A child with a prosthetic aortic valve who presents


with fevers, night sweats, and fatigue should be
treated empirically with ampicillin-sulbactam and
gentamicin.

41 CHALLENGE US NEXT QUESTION

Dashboard My Reports My Schedule Help About Contact Us Search Learning technology k area 9 Sign Out

NEJM Knowledge+ is a product of NEJM Group, a division of the Massachusetts Medical Society. Copyright ©2020 Massachusetts Medical Society. All rights reserved.
myknowledgeplus.nejm.org 0 1-1) Lr
NEJM Knowledge+ N Pediatrics Board Review

(REJM
Knowledge+ Pediatrics Board Review El
Sync
M

1
Which one of the following antibiotic regimens is the most appropriate empiric
therapy for an 8-year-old boy with a prosthetic aortic valve placed 6 years ago
who presents with high fever; night sweats; fatigue; diminished appetite; small
erythematous, nontender, nonblanching lesions on the hands; and a new
partial dehiscence of the aortic valve on echocardiogram?

15% Ceftriaxone

1% Ciprofloxacin

18% Piperacillin-tazobactam

63% Ampicillin-sulbactam and gentamicin

2% Levofloxacin

KEY LEARNING POINT


A child with a prosthetic aortic valve who presents
with fevers, night sweats, and fatigue should be
treated empirically with ampicillin-sulbactam and
gentamicin.
1u3
41 CHALLENGE US NEXT QUESTION

Dashboard My Reports My Schedule Help About Contact Us Search Learning technology k area 9 Sign Out

NEJM Knowledge+ is a product of NEJM Group, a division of the Massachusetts Medical Society. Copyright ©2020 Massachusetts Medical Society. All rights reserved.
myknowledgeplus.nejm.org 0 1-1) Lr
NEJM Knowledge+ (r Pediatrics Board Review

(REJM
Knowledge+ Pediatrics Board Review El
Sync
M

YOUR ANSWER IS CORRECT I I 100%


Which one of the following tests is most appropriate for establishing the
diagnosis in a 9-year-old girl with a prolonged course of fevers (including a
current fever of 39.1°C) who has vascular and immunologic phenomena that
raise concern for infective endocarditis?

0% Urine culture

95% Blood culture

1% Respiratory viral panel

2% B-type natriuretic peptide test

2% Troponin testing

KEY LEARNING POINT


The most appropriate diagnostic test for a child with
a prolonged course of fevers who has vascular and
i mmunologic phenomena that raise concern for
infective endocarditis is a blood culture in addition
to cardiac imaging. 1u3
41 CHALLENGE US NEXT QUESTION

Dashboard My Reports My Schedule Help About Contact Us Search Learning technology k area 9 Sign Out

NEJM Knowledge+ is a product of NEJM Group, a division of the Massachusetts Medical Society. Copyright ©2020 Massachusetts Medical Society. All rights reserved.
myknowledgeplus.nejm.org

NEJM Knowledge+ N Pediatrics Board Review

EJM
Knowledge+ Pediatrics Board Reviews
smc

Which one of the following interventions is most critical for a 2-day-old,


normotensive female infant who has difficulty feeding, becomes tachypneic
when she cries, appears cyanotic, and has moist mucous membranes, a
capillary refill time of 2 seconds, and a systolic 2/6 ejection murmur heard
best at the left upper sternal border?

95% Initiate prostaglandin infusion

0% Administer a dextrose bolus

0% Administer a dose of hydrocortisone

3% Administer a normal saline bolus

1% Administer ampicillin and gentamicin

The most appropriate intervention to prevent clinical


deterioration in a newborn with cyanosis,
tachypnea, and diminished oxygen saturation is
prostaglandin to maintain patency of the ductus
arteriosus.

RESOURCES

41 CHALLENGE US NEXT QUESTION

Dashboard My Reports My Schedule Help About Contact Us Search Learning technology 1 area9 Sign Out

NEJM Knowledge+ is a product of NEJM Group, a division of the Massachusetts Medical Society. Copyright ©2020 Massachusetts Medical Society. All rights reserved.
myknowledgeplus.nejm.org

NEJM Knowledge+ N Pediatrics Board Review

EJM
Knowledge+ Pediatrics Board Reviews
smc

YOUR ANSWER IS CORRECT 100%

What is the most appropriate next step for a full-term, 12-hour-old male infant
with bluish skin discoloration that is confined to his hands and feet, normal
findings on lung and cardiac examination, and a normal oxygen saturation?

2% Chest radiograph

0% Electrocardiogram

0% Oxygen supplementation via nasal cannula

2% Echocardiogram

95% [Continued observation without intervention

KEY LEARNING POINT mmill


Bluish discoloration of only the extremities in a
neonate is consistent with acrocyanosis.

41 CHALLENGE US NEXT QUESTION

Dashboard My Reports My Schedule Help About Contact Us Search Learning technology 1 area9 Sign Out

NEJM Knowledge+ is a 'product of NEJM Group, a division of the Massachusetts Medical Society. Copyright ©2020 Massachusetts Medical Society. All rights reserved.
myknowledgeplus.nejm.org O rh
NEJM Knowledge+ N Pediatrics Board Review

EJM
Knowledge+ Pediatrics Board Reviews
Sync

x YOUR ANSWER IS INCORRECT ■ 100%


A previously healthy, 3-month-old girl is brought to the emergency
department by her parents, who report that she has been irritable for 3 Which one of the following diagnoses is most likely in this case?
days, "breathing really fast," and tiring quickly while being breastfed.
She has not had any fevers, cough, congestion, vomiting, diarrhea, or
known exposure to sick contacts. She had an unremarkable birth
0% Viral bronchiolitis
history, was born full-term, and passed her neonatal screen for critical
congenital heart disease. There is no family history of congenital heart
77% Heart failure related to dilated cardiomyopathy
disease.
1% Nephrotic syndrome
The patient is afebrile and has a heart rate of 190 beats per minute, a
blood pressure of 60/45 mm Hg, a respiratory rate of 70 breaths per 2% Heart failure related to septic shock
minute, and an oxygen saturation of 94% while breathing ambient air.
Her weight, length, and head circumference are all in the 80th 20% Heart failure related to infantile supraventricular tachycardia
percentile for her age and have been stable since birth. She appears
fussy and slightly diaphoretic. Her cardiac examination is notable for a
2/6 blowing holosystolic murmur, loudest at the apex with radiation to
the left axilla, and for an S3 gallop. She has intercostal and subcostal
retractions with faint crackles heard at both lung bases. Her liver edge I he most likely diagnosis in a previously well,
is 4 cm below the right costal margin. Her distal extremities are cool to afebrile infant with signs and symptoms of heart
the touch with 4- to 5-second capillary refill. She has 1+ peripheral failure, evidence of cardiomegaly on chest
edema and 1+ periorbital edema. radiograph, and an electrocardiogram showing
sinus tachycardia and diffuse repolarization
Laboratory findings are as follows: abnormalities without evidence of ischemia is
Patient value Reference range dilated cardiomyopathy. RESOURCES
Serum creatinine 0.8 0.2-0.4
( mg/dL)
Aspartate 98 9-80 CHALLENGE US NEXT QUESTION
aminotransferase

Dashboard My Reports My Schedule Help About Contact Us Search Learning technology 1 area9 Sign Out

NEJM Knowledge+ is a product of NEJM Group, a division of the Massachusetts Medical Society. Copyright ©2020 Massachusetts Medical Society. All rights reserved.
myknowledgeplus.nejm.org 0 ri\-1 Lr
NEJM Knowledge+ 1,(-1\:f Pediatrics Board Review

(REJM
Knowledge+ Pediatrics Board Review El
Sync
Th

YOUR ANSWER IS INCORRECT ■ 100%

Which one of the following diagnoses is most likely in this case?

0% Viral bronchiolitis

77% Heart failure related to dilated cardiomyopathy

1% Nephrotic syndrome

2% Heart failure related to septic shock

20% Heart failure related to infantile supraventricular tachycardia

KEY LEARNING POINT im


The most likely diagnosis in a previously well,
afebrile infant with signs and symptoms of heart
failure, evidence of cardiomegaly on chest
radiograph, and an electrocardiogram showing
sinus tachycardia and diffuse repolarization
abnormalities without evidence of ischemia is
dilated cardiomyopathy. RESOURCES

Click to zoom

CHALLENGE US NEXT QUESTION

Dashboard My Reports My Schedule Help About Contact Us Search Learning technology k area Sign Out

NEJM Knowledge+ is a product of NEJM Group, a division of the Massachusetts Medical Society. Copyright ©2020 Massachusetts Medical Society. All rights reserved.
myknowledgeplus.nejm.org

NEJM Knowledge+ N Pediatrics Board Review

EJM
Knowledge+ Pediatrics Board Reviews
smc

..11. YOUR ANSWER IS CORRECT 85%

After initiation of prostaglandin El, what is the most appropriate next step for
a 9-day-old boy with severe pulmonary valve stenosis?

14% Initiation of sildenafil for pulmonary vasodilation

7°A Placement of a stent in the patent ductus arteriosus

14% Surgical placement of a systemic-to-pulmonary artery shunt

61% Balloon pulmonary valvuloplasty

2°A Initiation of furosemide

KEY LEARNING POINT —IMIN

Treatment for an infant with severe pulmonary valve


stenosis involves initiation of prostaglandin El
followed by balloon pulmonary valvuloplasty.

RESOURCES

CHALLENGE US NEXT QUESTION

Dashboard My Reports My Schedule Help About Contact Us Search Learning technology 1 area9 Sign Out

NEJM Knowledge+ is a product of NEJM Group, a division of the Massachusetts Medical Society. Copyright ©2020 Massachusetts Medical Society. All rights reserved.
myknowledgeplus.nejm.org 0 1-1) Lr
NEJM Knowledge+ N Pediatrics Board Review

(REJM
Knowledge+ Pediatrics Board Review El
Sync
M

YOUR ANSWER IS CORRECT ■ I 85%


After initiation of prostaglandin El, what is the most appropriate next step for
a 9-day-old boy with severe pulmonary valve stenosis?

14% Initiation of sildenafil for pulmonary vasodilation

7% Placement of a stent in the patent ductus arteriosus

14% Surgical placement of a systemic-to-pulmonary artery shunt

61% Balloon pulmonary valvuloplasty

2% Initiation of furosemide

Treatment for an infant with severe pulmonary valve


stenosis involves initiation of prostaglandin El
followed by balloon pulmonary valvuloplasty.

RESOURCES

41 CHALLENGE US NEXT QUESTION

Dashboard My Reports My Schedule Help About Contact Us Search Learning terd— /logv area 9 Sign Out

NEJM Knowledge+ is a product of NEJM Group, a division of the Massachusetts Medical Society. Copyright ©2020 Massachusetts Medical Society. All rights reserved.
myknowledgeplus.nejm.org 0 1-1) Lr
NEJM Knowledge+ N Pediatrics Board Review

(REJM
Knowledge+ Pediatrics Board Reviews
Sync
M

YOUR ANSWER IS CORRECT I 87%


Which one of the following tests would be most likely to reveal the correct
diagnosis in an afebrile 9-year-old boy who presents with tachycardia (145
beats per minute), abdominal pain, and a new cardiac murmur after a recent
viral illness with fevers?

6% Chest radiograph

Abdominal radiograph

0% Upper gastrointestinal radiograph series

1% Abdominal ultrasound

93% [Echocardiogram

Mow KEY LEARNING POINT


The combination of tachycardia, abdominal pain,
and a new cardiac murmur in a child with a recent
history of fevers should raise concern for postviral
myocarditis.

41 CHALLENGE US NEXT QUESTION

Dashboard My Reports My Schedule Help About Contact Us Search Learning terd— /logv area 9 Sign Out

NEJM Knowledge+ is a product of NEJM Group, a division of the Massachusetts Medical Society. Copyright ©2020 Massachusetts Medical Society. All rights reserved.
p

myKnowieagepius.nejm.org

NEJM Knowledge+ Pediatrics Board Review

novvieage • Pediatrics Board Reviews


Sync

Before diagnosing this patient with hypertension, which one of the following
steps should be taken?
A 7-year-old boy with no significant medical history o current health
concerns presents for routine medical care. His weig ht is in the 90th
percentile for his age and sex, his height is in the 25 ti percentile, and
Repeat the BP measurement in the right arm using a cuff whose bladder
his BMI is at >95th percentile. 50%
length is 80% to 100% of the patient's arm circumference
His current temperature is 37.8°C, his heart rate is 9 5 beats per
minute, and his respiratory rate is 22 breaths per min ute. His blood Measure the child's BP i n the left arm using a stethoscope and manual
36%
pressure (BP) in his right arm is 109/80 mm Hg (syst )lic BP at the 90th sphygmomanometer
percentile and diastolic BP at >95th percentile for ag sex, and
height). Repeat the BP measurement in the right arm, but use Korotkoff sounds
4%
1 and 4 to identify the systolic and diastolic pressures

Reassure the family that the patient is experiencing white-coat


2%
hypertension

Repeat the BP measurement in the right arm 2 to 3 minutes after the


7%
child takes a reiciAllis walk with a parent

When obtaining accurate BP measurements in the


pediatric population, the appropriate cuff size
should be based on the child's arm circumference.
11
CHALLENGE US NEXT QUESTION

ihboard M Re • orts M Schedule Hel About Contact Us Search area)


myknowledgeplus.nejm.org

NEJM Knowledge+ 'NN Pediatrics Board Review

EJM
Knowledge+ Pediatrics Board Reviews
Sync_

DETAILED FEEDBACK

A 7-year-old boy with no significant medical history or current health The 2017 hypertension guidelines from the American Academy of Pediatrics
concerns presents for routine medical care. His weight is in the 90th describe a standardized technique for obtaining blood pressure (BP)
percentile for his age and sex, his height is in the 25th percentile, and readings in children:
his BMI is at >95th percentile.
• BP measurements should be taken in the right arm for consistency and for
His current temperature is 37.8°C, his heart rate is 95 beats per comparison with values in standardized tables. Measurements in the left
minute, and his respiratory rate is 22 breaths per minute. His blood arm may be artificially low in the presence of coarctation of the aorta. The
pressure (BP) in his right arm is 109/80 mm Hg (systolic BP at the 90th arm should be uncovered and supported at the level of the heart.
percentile and diastolic BP at >95th percentile for age, sex, and • Children should be seated for 3 to 5 minutes before BP measurement,
height). with their back supported and their feet on the floor. BP should not be
measured after activity, including walks.
• The bladder length of the cuff should be 80% to 100% of the
circumference of the patient's arm, and the width should be at least 40%.
• In children, Korotkoff sound 1 is correlated to systolic BP, and Korotkoff
sound 5 is correlated to diastolic BP. Using Korotkoff sound 4 for BP
measurement is therefore not appropriate.
Notably, some children experience anxiety in a doctor's office, resulting in
white-coat hypertension. If a child's BP is borderline elevated in the office,
repeat values can be obtained by the school nurse, or ambulatory BP
monitoring can be ordered. Dismissing elevated BPs without follow-up is not
appropriate.

Last reviewed Oct 2019. Last modified Oct 2019.

BACK

Dashboard My Reports My Schedule Help About Contact Us Search Learning technology 1 area9 Sign Out

NEJM Knowledge+ is a product of NEJM Group, a division of the Massachusetts Medical Society. Copyright ©2020 Massachusetts Medical Society. All rights reserved.
myknowledgeplus.nejm.org

NEJM Knowledge+ N Pediatrics Board Review

EJM
Knowledge+ Pediatrics Board Reviews
smc

YOUR ANSWER IS CORRECT 80%

After administration of appropriate anti-streptococcal antibiotics, what is the


most appropriate initial treatment for an afebrile 12-year-old girl who presents
to the emergency department (ED) with acute rheumatic fever with migratory
polyarthritis and mild carditis?

Vancomycin and gentamicin

o, Methotrexate

28% No additional treatment is necessary

31% Prednisone with taper

39% (High-dose aspirin

I KEY LEARNING POINT


In a patient with acute rheumatic fever with
migratory polyarthritis and mild carditis, the most
appropriate initial treatment after administration of
appropriate antibiotics is high-dose aspirin.

CHALLENGE US NEXT QUESTION

Dashboard My Reports My Schedule Help About Contact Us Search Learning technology 1 area9 Sign Out

NEJM Knowledge+ is a product of NEJM Group, a division of the Massachusetts Medical Society. Copyright ©2020 Massachusetts Medical Society. All rights reserved.
myknowledgeplus.nejm.org 0 1-1) Lr
NEJM Knowledge+ (r Pediatrics Board Review

(REJM
Knowledge+ Pediatrics Board Review El
Sync
M

What is the most likely diagnosis in a previously healthy 9-year-old girl who has
chronic, progressive exercise intolerance; dyspnea on exertion; a single, loud
second heart sound without respiratory variation; a 2/6 blowing holosystolic
murmur, loudest at the left lower sternal border; a chest radiograph showing
cardiomegaly with enlargement of the main pulmonary artery and
noncongested lung fields; and an electrocardiogram that demonstrates right
axis deviation, right ventricular hypertrophy, and right atrial enlargement?

21% Partial anomalous pulmonary venous return

1% Pulmonary embolism

1% Asthma

66% Pulmonary arterial hypertension

12% Dilated cardiomyopathy

KEY LEARNING POINT .4=


The most li kely diagnosis in a school-age child with
exercise intolerance and dyspnea on exertion, a
single and loud second heart sound, and a
noncongested chest radiograph with enlargement of
the main pulmonary artery is pulmonary arterial

41 CHALLENGE US NEXT QUESTION

Dashboard My Reports My Schedule Help About Contact Us Search Learning technology k area 9 Sign Out

NEJM Knowledge+ is a product of NEJM Group, a division of the Massachusetts Medical Society. Copyright ©2020 Massachusetts Medical Society. All rights reserved.
myknowledgeplus.nejm.org O
NEJM Knowledge+ (I Pediatrics Board Review

Which one of the following pathogens is most likely to be implicated in the


case of a 4-year-old boy who presents to the emergency department with
tachypnea and tachycardia after 4 days of fever, loose stools, and fussiness;
who is found to have prolonged QT inten /al and ST-segment elevation on
electrocardiogram; and whose laboratory/ testing shows elevated levels of
inflammatory and cardiac biomarkers?

1% Haemophilus influenzaetype b

3% Staphylococcus aureus

9% rypanosoma cruzi

77% [Enterovirus

9% Influenza A

KEY LEARNING POINT

The pathogen most likely to cause acute myocarditis


in children is an enterovirus.

na
A

CHALLENGE US NEXT QUESTION


myknowledgeplus.nejm.org

NEJM Knowledge+ 0\1.( Pediatrics Board Review

EJM
Knowledge+ Pediatrics Board Reviews
smc

•MIL YOUR ANSWER IS CORRECT 84%

Which one of the following tests is most likely to confirm the correct diagnosis
in a 5-year-old boy with no significant medical history who presents with fever,
malaise, a pericardial friction rub, and chest pain that is worse with inspiration
but improves with upright positioning?

89% Electrocardiography

3% Antinuclear antibody testing

0% Lateral neck radiography

4% CT angiography

4% Troponin testing

The most useful diagnostic test in a patient with


suspected pericarditis is an electrocardiogram.

41 CHALLENGE US NEXT QUESTION

Dashboard My Reports My Schedule Help About Contact Us Search Learning technology 1 area9 Sign Out

NEJM Knowledge+ is a product of NEJM Group, a division of the Massachusetts Medical Society. Copyright ©2020 Massachusetts Medical Society. All rights reserved.
myknowledgeplus.nejm.org 0 1-1) Lr
NEJM Knowledge+ (r Pediatrics Board Review

(REJM
Knowledge+ Pediatrics Board Review El
Sync
M

Which one of the following medications is most important to initiate in a


15-year-old girl with a history of flulike symptoms and recent airplane travel
who presents to the emergency department with shortness of breath and
sharp chest pain that improves with leaning forward; whose physical
examination reveals poor breath sounds related to shallow breathing, a
normal first heart sound, a split second heart sound, a 2/6 vibratory systolic
murmur, and a rub; whose chest radiograph is normal; and whose
electrocardiogram reveals diffuse ST-segment elevations?

10% Anticoagulant

84% Nonsteroidal antiinflammatory drug

3% Glucocorticoid

5% Nitroglycerin

0% Proton-pump inhibitor

KEY LEARNING POINT

Appropriate first-line pharmacologic treatment for


acute viral pericarditis consists of a nonsteroidal
antiinflammatory drug.

41 CHALLENGE US NEXT QUESTION

Dashboard My Reports My Schedule Help About Contact Us Search Learning technology k area 9 Sign Out

NEJM Knowledge+ is a product of NEJM Group, a division of the Massachusetts Medical Society. Copyright ©2020 Massachusetts Medical Society. All rights reserved.
myknowledgeplus.nejm.org 0 1-1) Lr
NEJM Knowledge+ N Pediatrics Board Review

(REJM
Knowledge+ Pediatrics Board ReviewEl
Sync
M

YOUR ANSWER IS CORRECT

A full-term, 12-hour-old male infant whose size is appropriate for Which one of the following cardiac lesions is most likely to explain this
gestational age is evaluated in the well-baby nursery after a nurse patient's constellation of findings?
reports that he appears cyanotic. A pulse oximeter documents an
oxygen saturation of 83% in both the right upper and one lower
extremity. On examination, he has tachypnea without notable nasal
flaring, grunting, or intercostal retractions. He has no murmurs or other 25% Truncus arteriosus

notable findings on cardiac auscultation.


65% Transposition of the great arteries

3% Ventricular septal defect

1% Atrial septal defect

6% Aortic coarctation

The type of congenital heart disease most likely to


be present in a full-term male neonate who presents
with cyanosis, tachypnea, and normal size for
gestational age is transposition of the great arteries.

RESOURCES

CHALLENGE US NEXT QUESTION

Dashboard My Reports My Schedule Help About Contact Us Search Learning technology k area 9 Sign Out

NEJM Knowledge+ is a product of NEJM Group, a division of the Massachusetts Medical Society. Copyright ©2020 Massachusetts Medical Society. All rights reserved.
myknowledgeplus.nejm.org 0 1-1) Lr
NEJM Knowledge+ N Pediatrics Board Review

(REJM
Knowledge+ Pediatrics Board Review El
Sync
M

YOUR ANSWER IS INCORRECT I-


A 5-year-old boy presents with progressive muscle weakness and is found to
have a positive Gower sign, a normal cardiac examination, and a creatine
kinase level of 17,000 U/liter (reference range, 20-200). An
electrocardiogram and echocardiogram should be ordered to evaluate for
which one of the following conditions in this patient?

5% Aortic insufficiency

32% Hypertrophic cardiomyopathy

6% Arrhythmias

5% Mitral valve prolapse

52% Left ventricular dysfunction

A mlbw KEY LEARNING POINT


In patients with Duchenne muscular dystrophy,
echocardiograms are required at diagnosis and
serially over time to evaluate for left ventricular
dysfunction.

41 CHALLENGE US NEXT QUESTION

Dashboard My Reports My Schedule Help About Contact Us Search Learning technology k area 9 Sign Out

NEJM Knowledge+ is a product of NEJM Group, a division of the Massachusetts Medical Society. Copyright ©2020 Massachusetts Medical Society. All rights reserved.
myknowledgeplus.nejm.org 0 ri\-1 Lr
NEJM Knowledge+ 1,(-1\:f Pediatrics Board Review

(REJM
Knowledge+ Pediatrics Board Reviews
Sync
Th

YOUR ANSWER IS CORRECT OM I 82%


Which one of the following medications is most appropriate for a neonate with
a cyanotic heart lesion that is dependent on a patent ductus arteriosus for
pulmonary circulation?

100% Prostaglandin El

0% Epinephrine

0% Ceftriaxone

0% Milrinone

0% Furosemide

KEY LEARNING POINT J AM


In a neonate with cyanosis, poor weight gain, and a
suspected ductal-dependent cardiac lesion, it is
critical to initiate prostaglandin El.

CHALLENGE US NEXT QUESTION

Dashboard My Reports My Schedule Help About Contact Us Search Learning terd— /logv area 9 Sign Out

NEJM Knowledge+ is a product of NEJM Group, a division of the Massachusetts Medical Society. Copyright ©2020 Massachusetts Medical Society. All rights reserved.
myknowledgeplus.nejm.org 0 1-1) Lr
NEJM Knowledge+ N Pediatrics Board Review

(REJM
Knowledge+ Pediatrics Board Reviews
Sync
M

YOUR ANSWER IS CORRECT OM I 83%


Which one of the following recommendations is most appropriate for
endocarditis prophylaxis before dental cleaning in a 14-year-old girl with an
uncorrected cyanotic congenital heart defect?

5% Prescribe ampicillin 2 g intravenously 4 hours before the procedure

70% Prescribe amoxicillin 2 g by mouth one hour before the procedure

3% Prescribe clindamycin 600 mg by mouth the night before the procedure

4% Prescribe cephalexin 2 g by mouth the morning of the procedure

18% Proceed without antibiotic prophylaxis

When indicated, the antibiotic of choice for the


prevention of endocarditis in a patient having a
dental procedure is amoxicillin.

41 CHALLENGE US NEXT QUESTION

Dashboard My Reports My Schedule Help About Contact Us Search Learning terd— /logv area 9 Sign Out

NEJM Knowledge+ is a product of NEJM Group, a division of the Massachusetts Medical Society. Copyright ©2020 Massachusetts Medical Society. All rights reserved.
myknowledgeplus.nejm.org 0 1-1) Lr
NEJM Knowledge+ N Pediatrics Board Review

(REJM
Knowledge+ Pediatrics Board ReviewEl
Sync
M

A 5-year-old girl, who has been growing and developing normally, has
asymptomatic upper-extremity hypertension (127/64 mm Hg in the right arm
and 112/60 mm Hg in the right leg), diminished femoral pulses, and chest
radiograph showing inferior rib notching. For which one of the following
conditions is she at increased risk?

58% Intracranial aneurysm

13% Polycystic kidney disease

12% Hyperaldosteronism

2% Pheochromocytoma

15% Ehlers-Danlos syndrome

Coarctation of the aorta — evident in a patient


presenting with hypertension, diminished femoral
pulses, and radiographic evidence of rib notching —
increases the risk for intracranial aneurysm.

41 CHALLENGE US NEXT QUESTION

Dashboard My Reports My Schedule Help About Contact Us Search Learning technology k area 9 Sign Out

NEJM Knowledge+ is a product of NEJM Group, a division of the Massachusetts Medical Society. Copyright ©2020 Massachusetts Medical Society. All rights reserved.
myknowledgeplus.nejm.org

NEJM Knowledge+ N Pediatrics Board Review

EJM
Knowledge+ Pediatrics Board Reviews
smc

YOUR ANSWER IS CORRECT

An 11-year-old girl presents for her annual well-child visit. Her personal Which one of the following next steps is most appropriate to address this
and family medical history is unremarkable, and she has been child's elevated BP?
asymptomatic.

On arrival, her blood pressure (BP) is measured at 134/88 mm Hg via


an electronic BP device. Repeat measurement with an appropriately 1% Schedule an echocardiogram
sized manual cuff is 128/86 mm Hg, which falls in the range of stage 1
hypertension. Her growth parameters and physical examination are 4% Measure serum creatinine and electrolyte levels
normal.
Initiate a thiazide diuretic, and remeasure her BP in 4 weeks
Urinalysis reveals a specific gravity of 1.015 (reference range,
1.001-1.035), a pH of 6.0 (5.0-9.0), and negative findings for blood, 92% Arrange follow-up to repeat her BP measurement
protein, leukocytes, nitrites, ketones, and glucose.
2% Schedule a kidney ultrasound

In an asymptomatic school-age child with a first-


ti me elevated BP in the range of stage 1
hypertension, the most appropriate next step in
evaluation is to repeat her BP measurement at
follow-up.

RESOURCES

CHALLENGE US NEXT QUESTION

Dashboard My Reports My Schedule Help About Contact Us Search Learning technology 1 area9 Sign Out

NEJM Knowledge+ is a product of NEJM Group, a division of the Massachusetts Medical Society. Copyright ©2020 Massachusetts Medical Society. All rights reserved.
myknowledgeplus.nejm.org

NEJM Knowledge+ N Pediatrics Board Review

EJM
Knowledge+ Pediatrics Board Reviews
smc

What is the most appropriate immediate next step for a previously healthy
16-year- old girl who presents with dyspnea, abdominal pain, and nausea with
emesis; whose physical examination is notable for a temperature of 37.5°C, a
blood pressure of 110/74 mm Hg, cool extremities with a delayed capillary
refill, diffuse crackles on lung auscultation, and an audible third heart sound;
and whose chest radiograph shows cardiomegaly and increased vascular
markings?

1% Abdominal CT

7% Intravenous fluid bolus

78% Intravenous diuretic therapy

2% Intravenous antibiotics

11% Intravenous epinephrine

The mainstay of initial treatment for


decompensated heart failure is immediate
administration of intravenous diuretics.

E21
41 CHALLENGE US NEXT QUESTION

Dashboard My Reports My Schedule Help About Contact Us Search Learning technology 1 area9 Sign Out

NEJM Knowledge+ is a product of NEJM Group, a division of the Massachusetts Medical Society. Copyright ©2020 Massachusetts Medical Society. All rights reserved.
myknowledgeplus.nejm.org 0 1-1) Lr
NEJM Knowledge+ N Pediatrics Board Review

(REJM
Knowledge+ Pediatrics Board ReviewEl
Sync
M

YOUR ANSWER IS CORRECT

A 14-year-old boy presents to the emergency department (ED) because Which one of the following conditions is the most likely cause of this patient's
he feels like his heart is racing. He has been seen in the ED several sinus tachycardia?
ti mes during the past 4 months for palpitations that are becoming more
frequent and bothersome. He also says that he has had difficulty paying
attention in school lately, has been sleeping poorly, and often feels
overly warm. 0% Postural orthostatic tachycardia

He is afebrile and has a normal respiratory rate. His heart rate is 118 aJA Anxiety
beats per minute sitting and 125 beats per minute standing. His blood
pressure is 115/70 mm Hg sitting and 110/66 mm Hg standing. On 2''% Phenylpropanolamineoverdose
physical examination, he is thin but appears well. His precordium is
hyperdynamic with a midsystolic click, followed by a late 2/6 systolic 90% Hyperthyroidism
murmur heard best at the apex. His skin feels warm and sweaty. A
2% Coxsackie virus type B infection
slight tremor is evident when his hands are outstretched.

An electrocardiogram reveals sinus tachycardia.

Persistent sinus tachycardia associated with


changes in sleep and attention, a midsystolic click,
a tremor, and warm, sweaty skin should prompt
evaluation for hyperthyroidism.

RESOURCES

CHALLENGE US NEXT QUESTION

Dashboard My Reports My Schedule Help About Contact Us Search Learning technology k area 9 Sign Out

NEJM Knowledge+ is a product of NEJM Group, a division of the Massachusetts Medical Society. Copyright ©2020 Massachusetts Medical Society. All rights reserved.
myknowledgeplus.nejm.org

NEJM Knowledge+ N Pediatrics Board Review

EJM
Knowledge+ Pediatrics Board Reviews
smc

V mi .&
YOUR ANSWER IS CORRECT 86%

In addition to obtaining a personal and family history and performing a


physical examination, what testing is appropriate for a young adult who is
about to start participating in competitive sports?

Electrocardiogram and echocardiogram

1% Echocardiogram

1% Exercise stress test

2% Electrocardiogram, echocardiogram, and exercise stress test

94% [No further testing

KEY LEARNING POINT

The most appropriate initial screening in a young


adult before participation in competitive sports is a
complete personal history, family history, and
physical examination.

CHALLENGE US NEXT QUESTION

Dashboard My Reports My Schedule Help About Contact Us Search Learning technology 1 area9 Sign Out

NEJM Knowledge+ is a product of NEJM Group, a division of the Massachusetts Medical Society. Copyright ©2020 Massachusetts Medical Society. All rights reserved.
myknowledgeplus.nejm.org

NEJM Knowledge+ N Pediatrics Board Review

EJM
Knowledge+ Pediatrics Board Reviews
smc

YOUR ANSWER IS INCORRECT 87%

An 11-year-old boy reports that he was standing talking with friends Which one of the following next steps is most appropriate in evaluating this
when he started to feel dizzy and saw "spots" in his field of vision. He child?
briefly lost consciousness and started to fall down, but he was caught
by a classmate. He did not hit his head and was carefully laid on the
ground. He "woke up" within seconds, after which he felt well and acted
normally. 2/ Echocardiogram

He has not had any recent illnesses, he has no medical history, and 2Y0 Electroencephalogram
there is no family history of early cardiac or sudden death.
31% Reassurance
His vital signs are age-appropriate. His heart rate is regular with no
murmurs, rubs, or gallops; he has normal first and second heart 5% Tilt-table testing
sounds; and he has good pulses and perfusion. No focal neurologic
deficits are evident. The rest of his examination is within normal limits. 59% Electrocardiogram

In a child with an episode of uncomplicated syncope


who appears well, has no previous medical
problems, and has an unremarkable physical
examination, the most appropriate next step in
evaluation is an electrocardiogram.

RESOURCES

CHALLENGE US NEXT QUESTION

Dashboard My Reports My Schedule Help About Contact Us Search Learning technology 1 area9 Sign Out

NEJM Knowledge+ is a 'product of NEJM Group, a division of the Massachusetts Medical Society. Copyright ©2020 Massachusetts Medical Society. All rights reserved.
myknowledgeplus.nejm.org 0 1-1) Lr
NEJM Knowledge+ (r Pediatrics Board Review

(REJM
Knowledge+ Pediatrics Board ReviewEl
Sync
M

x YOUR ANSWER IS INCORRECT 84%

A 17-year-old female basketball player with no known medical history After the AED shock is discharged, which one of the following next steps is
collapses on the court during a game. She is found to be unconscious most appropriate?
and unresponsive, is not breathing, and lacks a pulse.

Bystanders immediately call for help, and cardiopulmonary


resuscitation is initiated. When an automated external defibrillator Check for a pulse; if none is present, resume cardiopulmonary
24%
(AED) is applied, it recommends a shock. resuscitation

Resume cardiopulmonary resuscitation for 2 minutes, and reanalyze the


71%
heart rhythm

Perform a blind finger sweep to ensure that no foreign bodies are


0%
obstructing the airway

Continue rescue breathing without chest compressions until the AED


2%
reanalyzes the rhythm

Wait for the AED to reanalyze the rhythm, and deliver a total of three
2%
stacked shocks if indicated

In a witnessed, out-of-hospital cardiac arrest during


a sporting event, the most appropriate action after

CHALLENGE US NEXT QUESTION

Dashboard My Reports My Schedule Help About Contact Us Search Learning technology k area 9 Sign Out

NEJM Knowledge+ is a product of NEJM Group, a division of the Massachusetts Medical Society. Copyright ©2020 Massachusetts Medical Society. All rights reserved.
myknowledgeplus.nejm.org 0 1-1) Lr
NEJM Knowledge+ 1,(-1\:f Pediatrics Board Review

(REJM
Knowledge+ Pediatrics Board Review El
Sync
M

After the AED shock is discharged, which one of the following next steps is
most appropriate?

A 17-year-old female basketball player with no known medical history


collapses on the court during a game. She is found to be unconscious
and unresponsive, is not breathing, and lacks a pulse. Check for a pulse; if none is present, resume cardiopulmonary
24%
resuscitation
Bystanders immediately call for help, and cardiopulmonary
resuscitation is initiated. When an automated external defibrillator Resume cardiopulmonary resuscitation for 2 minutes, and reanalyze the
71%
(AED) is applied, it recommends a shock. heart rhythm

Perform a blind finger sweep to ensure that no foreign bodies are


0%
obstructing the airway

Continue rescue breathing without chest compressions until the AED


2%
reanalyzes the rhythm

Wait for the AED to reanalyze the rhythm, and deliver a total of three
2%
stacked shocks if indicated

KEY LEARNING POINT


In a witnessed, out-of-hospital cardiac arrest during
a sporting event, the most appropriate action after
an automated external defibrillator provides a shock
is to resume cardiopulmonary resuscitation.

CHALLENGE US NEXT QUESTION

Dashboard My Reports My Schedule Help About Contact Us Search Learning technology area9 Sign Out

NEJM Knowledge+ is a product of NEJM Group, a division of the Massachusetts Medical Society. Copyright ©2020 Massachusetts Medical Society. All rights reserved.
myknowledgeplus.nejm.org 0 1-1) Lr
NEJM Knowledge+ N Pediatrics Board Review

(REJM
Knowledge+ Pediatrics Board Review El
Sync
M

Which one of the following diagnoses is most likely for a 16-year-old boy who
experiences episodes of pallor, sweating, lightheadedness, and tunnel vision
that are typically triggered by stress and culminate in a temporary loss of
consciousness— but who has normal brain-imaging findings and no
epileptiform activity on electroencephalogram?

92% Vasovagal syncope

4% Migraine

2% Complex partial seizures

1% Juvenile myoclonic epilepsy

2% Hypoglycemia

KEY LEARNING POINT —


UM
The most likely diagnosis in an adolescent with
episodic lightheadedness, pallor, and sweating that
are typically triggered by stress and culminate in a
temporary loss of consciousness is vasovagal
syncope.

41 CHALLENGE US NEXT QUESTION

Dashboard My Reports My Schedule Help About Contact Us Search Learning technology k area 9 Sign Out

NEJM Knowledge+ is a product of NEJM Group, a division of the Massachusetts Medical Society. Copyright ©2020 Massachusetts Medical Society. All rights reserved.
• • myknowledgeplus.nejm.org

NEJM Knowledge+ N Pediatrics Board Review

EJM
Knowledge+ Pediatrics Board Reviews
smc

YOUR ANSWER IS CORRECT

A 7-year-old girl presents with intermittent fevers, poor appetite, and What is the most likely cause of this patient's symptoms?
new-onset fatigue. The fevers first started a week ago, when she
developed a cold and cough, and she has missed the past week of
school as a result. Her parents note that she seemed out of breath this
morning while walking up a flight of stairs. 3% Pulmonary parenchymal infection

On physical examination, she appears fatigued but is in no acute 97% Poor cardiac output
distress. She is currently afebrile. Her heart rate is 132 beats per
minute, her respiratory rate is 28 breaths per minute, and her blood 0% Bronchospasm
pressure is 78/42 mm Hg in the right upper extremity. She has an
oxygen saturation of 95% while she breathes ambient air. 0% Mucus plugging

On cardiac auscultation, soft first and second heart sounds are heard, 0% Pulmonary hemorrhage
with no murmurs. Pulses are diminished in all extremities. The patient
is tachypneic without any other signs of increased work of breathing.
Diffuse crackles are heard in all lung fields. The liver edge is 3 cm
below the costal margin, and the extremities are slightly cool to the KEY LEARNING POINT
touch.
The most likely cause of tachypnea, pulmonary
crackles, soft heart sounds, and poor vascular
perfusion is poor cardiac output and heart failure.

RESOURCES

CHALLENGE US NEXT QUESTION

Dashboard My Reports My Schedule Help About Contact Us Search Learning technology 1 area9 Sign Out

NEJM Knowledge+ is a product of NEJM Group, a division of the Massachusetts Medical Society. Copyright ©2020 Massachusetts Medical Society. All rights reserved.
myknowledgeplus.nejm.org 0 1-1) Lr
NEJM Knowledge+ N Pediatrics Board Review

(REJM
Knowledge+ Pediatrics Board Review El
Sync
M

YOUR ANSWER IS INCORRECT

A 16-year-old girl has had three separate blood pressure (BP) measurements
of roughly 130/80 mm Hg. What is the correct interpretation of her BP?

19% Elevated BP (previously described as "prehypertension")

2% Normal BP

66% Stage 1 hypertension

9% Uninterpretable in the current circumstances

4% Stage 2 hypertension

KEY LEARNING POINT

In children a.13 years of age, stage 1 hypertension


is diagnosed on the basis of repeated BP
measurements between 130/80 and 139/89 mm
Hg.

RESOURCES

41 CHALLENGE US NEXT QUESTION

Dashboard My Reports My Schedule Help About Contact Us Search Learning technology k area 9 Sign Out

NEJM Knowledge+ is a product of NEJM Group, a division of the Massachusetts Medical Society. Copyright ©2020 Massachusetts Medical Society. All rights reserved.
myknowledgeplus.nejm.org 0 1-1) Lr
NEJM Knowledge+ N Pediatrics Board Review

(REJM
Knowledge+ Pediatrics Board Review El
Sync
M

What is the most appropriate next step for an asymptomatic 15-year-old boy
presenting for a sports preparticipation evaluation whose father has newly
diagnosed hypertrophic cardiomyopathy and whose own resting
electrocardiogram and echocardiogram are both normal?

0% Referral for placement of an implantable cardioverter-defibrillator

Genetic testing of the father, followed by targeted genetic testing of the


58%
patient

0% Cardiac MRI

32% Repeat screening echocardiography every other year

10% Restriction from competitive sports

KEY LEARNING POINT MEW


The best next step in evaluation for a patient with a
family history of hypertrophic cardiomyopathy who
has a normal electrocardiogram and a normal
echocardiogram is genetic testing of the affected
family member, followed by targeted genetic testing
of the patient.
RESOURCES

41 CHALLENGE US NEXT QUESTION

Dashboard My Reports My Schedule Help About Contact Us Search Learning technology k area 9 Sign Out

NEJM Knowledge+ is a product of NEJM Group, a division of the Massachusetts Medical Society. Copyright ©2020 Massachusetts Medical Society. All rights reserved.
• • myknowledgeplus.nejm.org O rh
NEJM Knowledge+ N Pediatrics Board Review

EJM
Knowledge+ Pediatrics Board Reviews
smc

YOUR ANSWER IS CORRECT

In addition to performing a urinalysis and measuring the serum creatinine


A 10-year-old girl presents for follow-up several days after her blood
level, what is the most appropriate immediate next step in managing this
pressure (BP) was measured at 136/86 mm Hg during a routine
patient's elevated BP?
evaluation. A follow-up BP measurement by her school nurse was
recorded at 134/82 mm Hg. Since she was last seen, the patient has
had intermittent headaches and today reports a headache that she
rates as 6 out of 10 in intensity. 57% Refer to the emergency department

Her current BP, measured with an appropriately sized manual cuff, is 10% Measure the level of thyroid-stimulating hormone
142/90 mm Hg in both arms. She has a normal physical examination.
Measure plasma catecholamines, metanephrines, and plasma renin
23%
activity

1% Order a 24-hour urine collection for calcium, phosphate, and creatinine

9% Arrange follow-up within the next week to repeat the BP measurement

KEY LEARNING POINT

Children with stage 2 hypertension should be


referred to the emergency department if they are
symptomatic or their BP is >30 mm Hg above the
95th percentile (or >180/120 mm Hg in an
adolescent).

CHALLENGE US NEXT QUESTION

Dashboard My Reports My Schedule Help About Contact Us Search Learning technology 1 area9 Sign Out

NEJM Knowledge+ is a product of NEJM Group, a division of the Massachusetts Medical Society. Copyright ©2020 Massachusetts Medical Society. All rights reserved.
• • myknowledgeplus.nejm.org

NEJM Knowledge+ (Taf Pprliatrirg Rnarrl P2pvipw

EJM
Knowledge+ Pediatrics Board Reviews
Sync_

V = Elm YOUR ANSWER IS CORRECT 78%

Which one of the following treatments is the best next step in managing a
stable 14-year-old boy with palpitations and an electrocardiogram showing a
narrow-complex tachycardia of 162 beats per minute that does not respond to
vagal maneuvers?

3% Amiodarone

0% Diltiazem

93% (Adenosine

2% Metoprolol

1% Direct-current cardioversion

KEY LEARNING POINT

The most appropriate treatment for a


hemodynamically stable supraventricular
tachycardia that does not respond to vagal
maneuvers is intravenous adenosine.

41 CHALLENGE US NEXT QUESTION

Dashboard My Reports My Schedule Help About Contact Us Search Learning technology 1 area9 Sign Out

NEJM Knowledge+ is a product of NEJM Group, a division of the Massachusetts Medical Society. Copyright ©2020 Massachusetts Medical Society. All rights reserved.
myknowledgeplus.nejm.org 0 1-1) Lr
NEJM Knowledge+ (r Pediatrics Board Review

(REJM
Knowledge+ Pediatrics Board Review El
Sync
M

YOUR ANSWER IS CORRECT

Which one of the following diagnoses is most likely for a 9-year-old boy with a
history of exercise intolerance who developed syncope while running, who has
normal cardiac and neurologic examinations, and whose electrocardiogram
reveals no apparent relationship between the P waves and QRS complexes?

4% Hypertrophic cardiomyopathy

2% Neurocardiogenic syncope

4% Brugada syndrome

88% Third-degree atrioventricular heart block

2% Wolff-Parkinson-White syndrome

KEY LEARNING POINT

A syncopal episode associated with exercise, chest


pain, or palpitations raises suspicion for a cardiac
cause.

1u3
41 CHALLENGE US NEXT QUESTION

Dashboard My Reports My Schedule Help About Contact Us Search Learning technology k area 9 Sign Out

NEJM Knowledge+ is a product of NEJM Group, a division of the Massachusetts Medical Society. Copyright ©2020 Massachusetts Medical Society. All rights reserved.
• • myknowledgeplus.nejm.org O rh
NEJM Knowledge+ (J\l.( Pediatrics Board Review

EJM
Knowledge+ Pediatrics Board Reviews
smc

o nlim YOUR ANSWER IS CORRECT


Which one of the following management approaches is most appropriate for a
14-year-old girl with a classic presentation of vasovagal syncope who has a
normal physical examination and a normal electrocardiogram?

91% Reassurance

2% Hotter monitor

5% Tilt-table study

2% Event recorder

1% Echocardiogram

KEY LEARNING POINT

The most appropriate next step for a patient with a


classic presentation of vasovagal syncope and a
normal electrocardiogram is reassurance.

CHALLENGE US NEXT QUESTION

Dashboard My Reports My Schedule Help About Contact Us Search Learning technology 1 area9 Sign Out

NEJM Knowledge+ is a product of NEJM Group, a division of the Massachusetts Medical Society. Copyright ©2020 Massachusetts Medical Society. All rights reserved.
myknowledgeplus.nejm.org 0 1-1) Lr
NEJM Knowledge+ 1,(-1\:f Pediatrics Board Review

(REJM
Knowledge+ Pediatrics Board Review El
Sync

In addition to close follow-up, which one of the following therapeutic strategies


is most appropriate for a 16-year-old boy with a BMI at the 98th percentile
who has a history of essential hypertension that was not responsive to 6
months of lifestyle interventions but has responded to losartan, who has a
normal comprehensive metabolic panel, and whose most recent fasting lipid
panel and glycated hemoglobin results (shown below) are improved but
remain abnormal?
Patient value Reference range
Glycated hemoglobin (%) 5.5 4.5-5.6
Total cholesterol (mg/dL) 209 <170
LDL cholesterol (mg/dL) 156 <110
HDL cholesterol (mg/dL) 32 >35
Triglycerides (mg/dL) 280 24-145
Glucose (mg/dL) 95 70-105
Thyroid-stimulating hormone (pU/mL) 1.2 0.5-4.0

33% Initiate atorvastatin

0% Initiate cholestyramine

3% Initiate metformin

5% Initiate gemfibrozil

59% Continue with lifestyle modifications alone

CHALLENGE US NEXT QUESTION

Dashboard My Reports My Schedule Help About Contact Us Search Learning terd— /logv area 9 Sign Out

NEJM Knowledge+ is a product of NEJM Group, a division of the Massachusetts Medical Society. Copyright ©2020 Massachusetts Medical Society. All rights reserved.
myknowledgeplus.nejm.org

NEJM Knowledge+ N Pediatrics Board Review

EJM
Knowledge+ Pediatrics Board Reviews
smc

Which one of the following conditions is the most likely underlying cause of
pleuritic chest pain (worse with lying down), exertional dyspnea, bilateral
pulmonary rales, diffuse ST-segment elevation with low voltage on
electrocardiogram, and evidence of pulmonary vascular congestion on chest
radiograph in a 17-year-old boy who had a recent viral illness?

72% Parvovirus B19 infection

4% Cocaine ingestion

9%
Streptococcus pneumoniae infection

7% Anomalous left coronary artery from the right coronary sinus

7% Inflammation of the costosternal junctions

KEY LEARNING POINT


I he most likely diagnosis in an adolescent who
presents with acute positional chest pain, exertional
dyspnea, a recent viral illness, diffuse ST-segment
elevation on electrocardiogram, and pulmonary
vascular congestion on chest radiograph is viral
perimyocarditis (most commonly due to parvovirus
B19, human herpesvirus 6, or enteroviruses such RESOURCES

41 CHALLENGE US NEXT QUESTION

Dashboard My Reports My Schedule Help About Contact Us Search Learning technology 1 area9 Sign Out

NEJM Knowledge+ is a product of NEJM Group, a division of the Massachusetts Medical Society. Copyright ©2020 Massachusetts Medical Society. All rights reserved.
myknowledgeplus.nejm.org 0 1-1) of

NEJM Knowledge+ (r Pediatrics Board Review

(REJM
Knowledge+ Pediatrics Board Review El
pc_
M

Which one of the following diagnoses is most likely in a 3-year-old child who
has recently been adopted internationally and is undergoing his first medical
evaluation in the United States, whose parents have observed exertional
dyspnea and episodes of cyanosis and squatting, and whose physical
examination reveals a weight at the 4th percentile for age and a harsh systolic
ejection murmur?

1% Truncus arteriosus

3% Patent ductus arteriosus

0% Transposition of the great arteries

0% Total anomalous pulmonary venous connection

96% Tetralogy of Fallot

KEY LEARNING POINT

The most likely diagnosis in a 3-year-old child with


exertional dyspnea, episodes of cyanosis and
squatting, and a systolic murmur is tetralogy of
Fallot.

41 CHALLENGE US NEXT QUESTION

Dashboard My Reports My Schedule Help About Contact Us Search Learning terd— /logv area 9 Sign Out

NEJM Knowledge+ is a product of NEJM Group, a division of the Massachusetts Medical Society. Copyright ©2020 Massachusetts Medical Society. All rights reserved.
myknowledgeplus.nejm.org 0 1-1) Lr
NEJM Knowledge+ (r Pediatrics Board Review

(REJM
Knowledge+ Pediatrics Board Review El
Sync
M

YOUR ANSWER IS CORRECT

A healthy 4-year-old girl who is growing and developing normally is Which one of the following next steps is most appropriate for this child?
found to have a heart murmur during her physical examination. She
has no family history of valvular heart disease or sudden cardiac death.

Her temperature is 36.7°C, her heart rate is 110 beats per minute, her 59% Reassure the parents
respiratory rate is 22 breaths per minute, and her blood pressure is
92/57 mm Hg. She has a II/VI high-pitched crescendo-decrescendo 4% Obtain an electrocardiogram and chest radiograph
midsystolic murmur best heard at the left sternal border between the
second and third intercostal spaces. The murmur is loudest when she is 28% Obtain an echocardiogram
supine. There are no clicks or gallops.
0/ Obtain a throat swab for rapid streptococcal antigen detection
The patient is warm and well perfused, and her peripheral pulses are
symmetric bilaterally. Her lungs are clear to auscultation. Arrange for a follow-up visit in 2 months to evaluate for any progression
10/
in the murmur

KEY LEARNING POINT

In a 4-year-old child with a pulmonary flow murmur


and no related historical features, physical
examination findings, or family history of cardiac
disease, the most appropriate next step is to
reassure the parents and provide them with
anticipatory guidance.
RESOURCES

CHALLENGE US NEXT QUESTION

Dashboard My Reports My Schedule Help About Contact Us Search Learning technology k area 9 Sign Out

NEJM Knowledge+ is a product of NEJM Group, a division of the Massachusetts Medical Society. Copyright ©2020 Massachusetts Medical Society. All rights reserved.
myknowledgeplus.nejm.org 0 1-1) Lr
NEJM Knowledge+ (r Pediatrics Board Review

(REJM
Knowledge+ Pediatrics Board Review El
Sync
M

What is the most important immediate management for a 15-year-old girl who
presents with palpitations, altered mental status, hypotension, and an
electrocardiogram that reveals a regular, narrow-complex tachycardia at a rate
of 211 beats per minute without visible P waves?

1% Administer intravenous amiodarone

20% Administer intravenous adenosine

61% Conduct synchronized cardioversion

3% Administer a fluid bolus

15% Instruct the patient to perform vagal maneuvers

KEY LEARNING POINT

The most important initial treatment for


supraventricular tachycardia in a patient who is
hemodynamically unstable (as evidenced by
hypotension, altered mental status, or signs of
shock), is synchronized cardioversion. col
RESOURCES

41 CHALLENGE US NEXT QUESTION

Dashboard My Reports My Schedule Help About Contact Us Search Learning technology k area 9 Sign Out

NEJM Knowledge+ is a product of NEJM Group, a division of the Massachusetts Medical Society. Copyright ©2020 Massachusetts Medical Society. All rights reserved.
• myknowledgeplus.nejm.org

NEJM Knowledge+ 0\1.( Pediatrics Board Review

EJM
Knowledge+ Pediatrics Board Reviews
Sync
From "myknowledgeplus.nejm.org ":
Are you sure you want to leave this page?
NSWER IS CORRECT 80%

A 6-month-old boy who was born full term pr€ vents curd well-child visit. dir gs is this patient most likely to have on
He has had several upper respiratory infectio s during the past few
months but has never required hospitalizatio . He is now at the 5t1 -
percentile for body weight, down from the 30tn percenute aunng fits
previous visit. His mother remarks that he seems to lose his breath very
easily and is often sweaty, even when drinking a bottle. 23% Decreased oxygen saturat i on

18% Diminished femoral pulses


On examination, he is tachypneic and has a loud continuous murmur
heard best at the left upper sternal border. 3% Cyanotic extremities

45% Bounding peripheral pulses

11% Elevated diastolic pressure

The physical examination of an infant with


significant left-to-right cardiac shunting often
reveals bounding peripheral pulses.

RESOURCES

CHALLENGE US NEXT QUESTION

Dashboard My Reports My Schedule Help About Contact Us Search Learning technology 1 area9 Sign Out

NEJM Knowledge+ is a product of NEJM Group, a division of the Massachusetts Medical Society. Copyright ©2020 Massachusetts Medical Society. All rights reserved.
myknowledgeplus.nejm.org 0 1-1) Lr
NEJM Knowledge+ (r Pediatrics Board Review

(REJM
Knowledge+ Pediatrics Board ReviewEl
Sync
M

A 21-year-old male college student presents to the emergency


department after experiencing a witnessed syncopal episode that
YOUR ANSWER IS CORRECT MEM 81%
lasted a few seconds while he was at soccer practice. He remembers
having dyspnea, palpitations, and chest pain before the episode. There What is the most likely cause of this patient's syncopal event?
was no injury or trauma. He reports no incontinence or confusion but
states he was a little tired after the incident.
2 `% Athletic heart syndrome
The patient appears comfortable. His heart rate is 68 beats per minute,
his respiratory rate is 12 breaths per minute, his blood pressure is 5`% Bicuspid aortic valve
132/84 mm Hg, and his oxygen saturation is 99% while he breathes
ambient air. He has normal first and second heart sounds with a 82 k Hypertrophic cardiomyopathy
crescendo-decrescendo systolic murmur, best heard at the left sternal
border, that increases with the Valsalva maneuver. 9% Long-QT syndrome

An electrocardiogram is obtained (figure). 3% Eisenmenger syndrome

KEY LEARNING POINT j

In a young man who has a systolic murmur that


=
0.1 V2 ! VS
worsens with the Valsalva maneuver and whose
electrocardiogram reveals left ventricular
hypertrophy and a strain pattern of downsloping ST-
"V V6
T changes and T-wave inversion in the anterolateral
leads, the most likely cause of an acute syncopal
event is hvoertroohic cardiomvooathv. RESOURCES

CHALLENGE US NEXT QUESTION

Dashboard My Reports My Schedule Help About Contact Us Search Learning technology k area 9 Sign Out

NEJM Knowledge+ is a product of NEJM Group, a division of the Massachusetts Medical Society. Copyright ©2020 Massachusetts Medical Society. All rights reserved.
myknowledgeplus.nejm.org

NEJM Knowledge+ N Pediatrics Board Review

EJM
Knowledge+ Pediatrics Board Reviews
smc

YOUR ANSWER IS CORRECT I 81%


A 9-year-old girl presents to the emergency department after 10 days of Which one of the following tests is most likely to establish the diagnosis in this
high fevers, diminished appetite, fatigue, and intermittent arm and leg patient?
pain. During the past few days, she has also had a rash on her hands
and feet.

On examination, she has a temperature of 39.1°C, a heart rate of 113 76% Blood culture
beats per minute, a respiratory rate of 20 breaths per minute, and a
blood pressure of 110/45 mm Hg. Since her last visit to her physician 3 4% B-type natriuretic peptide test
months ago, she has lost 1.5 kg. She has a new systolic murmur heard
best in the left lower border, her breath sounds are clear, and her 16% Respiratory viral panel
abdomen is soft and nondistended. She has nonblanching, circular
lesions on her hands and soles. 4% Troponin testing

Urinalysis reveals microscopic hematuria. Initial screening tests, 0% Urine culture


including a complete blood count and basic metabolic panel, are
ordered.

The most appropriate diagnostic test for a child with


a prolonged course of fevers who has vascular and
i mmunologic phenomena that raise concern for
infective endocarditis is a blood culture in addition
to cardiac imaging. 1ED
RESOURCES

CHALLENGE US NEXT QUESTION

Dashboard My Reports My Schedule Help About Contact Us Search Learning technology 1 area9 Sign Out

NEJM Knowledge+ is a product of NEJM Group, a division of the Massachusetts Medical Society. Copyright ©2020 Massachusetts Medical Society. All rights reserved.
myknowledgeplus.nejm.org 0 1-1) Lr
NEJM Knowledge+ (r Pediatrics Board Review

(REJM
Knowledge+ Pediatrics Board Review El
Sync
M

YOUR ANSWER IS CORRECT

An 8-year-old boy is brought to the emergency department by his Which one of the following antibiotic regimens is the best empiric therapy for
mother because of persistent fevers, night sweats, and diminished this patient?
appetite for the past week. He has also been coming home from school
tired and has missed several days of baseball practice. His history is
significant for aortic stenosis that was diagnosed at birth and repaired
surgically with placement of a prosthetic aortic valve when he was a 20% Ceftriaxone
toddler. Since the surgery, he has been growing and developing
normally.
61% Ampicillin-sulbactam and gentamicin

On physical examination, he has a temperature of 39.4°C, a heart rate 1% Ciprofloxacin


of 123 beats per minute, a respiratory rate of 20 breaths per minute,
and a blood pressure of 103/63 mm Hg. He appears fatigued but is 3% Levofloxacin
sitting comfortably on the bed. The first and second heart sounds are
16% Piperacillin-tazobactam
heard, as well as a systolic 3/4 harsh murmur that radiates to the neck.
His abdomen is nontender and nondistended, his splenic edge is 4 cm
below the costal margin, and his liver is not palpable. Small
erythematous, nontender, nonblanching lesions are evident on his
hands.

Blood cultures are ordered, and an echocardiogram is obtained that A child with a prosthetic aortic valve who presents
reveals a new partial dehiscence of the patient's aortic valve. with fevers, night sweats, and fatigue should be
Acetaminophen is given for the fever. treated empirically with ampicillin-sulbactam and
gentamicin.

RESOURCES

CHALLENGE US NEXT QUESTION

Dashboard My Reports My Schedule Help About Contact Us Search Learning technology area9 Sign Out

NEJM Knowledge+ is a product of NEJM Group, a division of the Massachusetts Medical Society. Copyright ©2020 Massachusetts Medical Society. All rights reserved.
myknowledgeplus.nejm.org

NEJM Knowledge+ N Pediatrics Board Review

EJM
Knowledge+ Pediatrics Board Reviews
smc

Before diagnosing this patient with hypertension, which one of the following
steps should be taken?

A 7-year-old boy with no significant medical history or current health


concerns presents for routine medical care. His weight is in the 90th Repeat the BP measurement in the right arm using a cuff whose bladder
percentile for his age and sex, his height is in the 25th percentile, and 50%
length is 80% to 100% of the patient's arm circumference
his BMI is at >95th percentile.

His current temperature is 37.8°C, his heart rate is 95 beats per Reassure the family that the patient is experiencing white-coat
minute, and his respiratory rate is 22 breaths per minute. His blood hypertension
pressure (BP) in his right arm is 109/80 mm Hg (systolic BP at the 90th
Measure the child's BP in the left arm using a stethoscope and manual
percentile and diastolic BP at >95th percentile for age, sex, and 36%
height). sphygmomanometer

Repeat the BP measurement in the right arm, but use Korotkoff sounds
4%
1 and 4 to identify the systolic and diastolic pressures

Repeat the BP measurement in the right arm 2 to 3 minutes after the


7%
child takes a relaxing walk with a parent

When obtaining accurate BP measurements in the


pediatric population, the appropriate cuff size
should be based on the child's arm circumference.

CHALLENGE US NEXT QUESTION

Dashboard My Reports My Schedule Help About Contact Us Search Learning technology 1 area9 Sign Out

NEJM Knowledge+ is a product of NEJM Group, a division of the Massachusetts Medical Society. Copyright ©2020 Massachusetts Medical Society. All rights reserved.
myknowledgeplus.nejm.org

NEJM Knowledge+ 'NN Pediatrics Board Review

EJM
Knowledge+ Pediatrics Board Reviews
Sync

YOUR ANSWER IS CORRECT U


A 9-day-old boy is brought in by his mother for concerns of poor feeding
and color changes. He was born full term via cesarean section for
Which one of the following next steps is most appropriate for this patient?
nonreassuring fetal heart tones, and his nursery stay was
unremarkable. However, since being home, he has been breastfeeding
poorly, with diminished energy, and has been taking longer to feed. His
mother states that he "gets blue when he cries." She has also noticed 8/ Initiation of furosemide
rapid breathing and occasional grunting.
15/ Placement of a stent in the patent ductus arteriosus

On physical examination, the patient has a heart rate of 165 beats per 24/ Surgical placement of a systemic-to-pulmonary artery shunt
minute, normal blood pressure for his age, and pre- and postductal
oxygen saturations of 70% and 68%, respectively, while breathing 14/ Initiation of sildenafil for pulmonary vasodilation
ambient air. He appears fatigued and cyanotic, and he is tachypneic
with subcostal retractions. A prominent apical impulse and a thrill are 39 0
Balloon pulmonary valvuloplasty
detected on palpation, and auscultation reveals a click just after the
first heart sound; a fixed, split second heart sound; and a 4/6
crescendo-decrescendo systolic murmur, heard loudest in late systole
extending into diastole, both at the left upper sternal border. The infant KEY LEARNING POINT
has normal bowel sounds, and his liver is palpable 2 cm below the
costal margin. Treatment for an infant with severe pulmonary valve
stenosis involves initiation of prostaglandin El
followed by balloon pulmonary valvuloplasty.
A chest radiograph shows normal heart size and reduced pulmonary
vascularity without focal consolidation. An electrocardiogram
demonstrates sinus tachycardia with right axis deviation, a pure R wave
1ED
in lead V1 that is >20 mm, and inverted T waves in lead V4. RESOURCES

Prostaglandin El is initiated.

CHALLENGE US NEXT QUESTION

Dashboard My Reports My Schedule Help About Contact Us Search Learning technology 1 area9 Sign Out

NEJM Knowledge+ is a product of NEJM Group, a division of the Massachusetts Medical Society. Copyright ©2020 Massachusetts Medical Society. All rights reserved.
myknowledgeplus.nejm.org 0 1-1) Lr
NEJM Knowledge+ (r Pediatrics Board Review

(REJM
Knowledge+ Pediatrics Board Review El
Sync
M

After the AED shock is discharged, which one of the following next steps is
most appropriate?

A 17-year-old female basketball player with no known medical history


collapses on the court during a game. She is found to be unconscious
and unresponsive, is not breathing, and lacks a pulse. Wait for the AED to reanalyze the rhythm, and deliver a total of three
2%
stacked shocks if indicated
Bystanders immediately call for help, and cardiopulmonary
resuscitation is initiated. When an automated external defibrillator Continue rescue breathing without chest compressions until the AED
2%
(AED) is applied, it recommends a shock. reanalyzes the rhythm

Perform a blind finger sweep to ensure that no foreign bodies are


0%
obstructing the airway

Resume cardiopulmonary resuscitation for 2 minutes, and reanalyze the


71%
heart rhythm

Check for a pulse; if none is present, resume cardiopulmonary


24%
resuscitation

In a witnessed, out-of-hospital cardiac arrest during


a sporting event, the most appropriate action after
an automated external defibrillator provides a shock
is to resume cardiopulmonary resuscitation.

CHALLENGE US NEXT QUESTION

Dashboard My Reports My Schedule Help About Contact Us Search Learning technology area9 Sign Out

NEJM Knowledge+ is a product of NEJM Group, a division of the Massachusetts Medical Society. Copyright ©2020 Massachusetts Medical Society. All rights reserved.
myknowledgeplus.nejm.org

NEJM Knowledge+ N Pediatrics Board Review

EJM
Knowledge+ Pediatrics Board Reviews
smc

YOUR ANSWER IS CORRECT MEM 86%


A full-term, 12-hour-old male infant is being evaluated in the newborn What is the most appropriate next step for this patient?
nursery for bluish discoloration of his hands and feet. His lips and
tongue appear pink. He weighs 3.0 kg and has a respiratory rate of 30
breaths per minute, a heart rate of 140 beats per minute, and a blood
pressure of 80/50 mm Hg. Oxygen saturation is 99% when measured 99% Continued observation without intervention

from the right hand and 98% from the left foot.
0% Chest radiograph

The patient's lungs are clear, and he exhibits no work of breathing or 0% Oxygen supplementation via nasal cannula
tachypnea. Peripheral pulses, including femoral pulses, are palpable
and 2+ with normal capillary refill. Cardiac examination is normal 1% Echocardiogram
without audible murmur.
0% Electrocardiogram

Bluish discoloration of only the extremities in a


neonate is consistent with acrocyanosis.

1ED
RESOURCES

CHALLENGE US NEXT QUESTION

Dashboard My Reports My Schedule Help About Contact Us Search Learning technology 1 area9 Sign Out

NEJM Knowledge+ is a product of NEJM Group, a division of the Massachusetts Medical Society. Copyright ©2020 Massachusetts Medical Society. All rights reserved.
myknowledgeplus.nejm.org 0 1-1) Lr
NEJM Knowledge+ (r Pediatrics Board Review

(REJM
Knowledge+ Pediatrics Board Review El
Sync
M

A previously healthy 15-year-old girl presents to the emergency YOUR ANSWER IS CORRECT
department with a 3-day history of sharp, left-sided, nonradiating chest
pain that worsens with inspiration and lying down — and improves with
Which one of the following medications is most important to initiate in this
sitting up and leaning forward. The patient also reports difficulty
breathing and catching her breath. She has not traveled internationally, patient?
but she did fly from Chicago to New York 2 weeks ago and developed
flulike symptoms (fever, rhinorrhea, and cough) around that time.
3% Anticoagulant
Physical examination reveals a heart rate of 110 beats per minute, a
blood pressure of 110/70 mm Hg, a respiratory rate of 18 breaths per 91% Nonsteroidal antiinflammatory drug
minute, and an oxygen saturation of 98% while the patient breathes
ambient air. Her breath sounds are difficult to auscultate because her 4% Nitroglycerin
pain is causing her to take shallow breaths. She has a normal first
heart sound, a split second heart sound, a 2/6 vibratory systolic 0% Proton-pump inhibitor
murmur heard at the left midsternal border, and a rub. Her chest pain
is not reproducible with palpation over costochondral junctions. She 2% Glucocorticoid
has no hepatomegaly or peripheral edema.

A chest radiograph is normal. An electrocardiogram is obtained (figure).

Appropriate first-line pharmacologic treatment for


acute viral pericarditis consists of a nonsteroidal
antiinflammatory drug.

RESOURCES

CHALLENGE US NEXT QUESTION

Dashboard My Reports My Schedule Help About Contact Us Search Learning technology area9 Sign Out

NEJM Knowledge+ is a product of NEJM Group, a division of the Massachusetts Medical Society. Copyright ©2020 Massachusetts Medical Society. All rights reserved.
myknowledgeplus.nejm.org O rh
NEJM Knowledge+ 'NN Pediatrics Board Review

EJM
Knowledge+ Pediatrics Board Reviews
Sync

A 4-year-old boy is admitted from the emergency department (ED) for YOUR ANSWER IS CORRECT
dehydration. Four days ago, he developed a fever (maximum
temperature, 38.9°C), fussiness, loose stools, and diminished oral
Which one of the following pathogens is the most likely cause of this patient's
intake. His mother was concerned that he had a sore throat because he
symptoms?
would cry whenever he tried to drink. In the ED, he was tachycardic, had
dry mucous membranes, and refused all oral fluids. Given concerns
about dehydration, he received a total fluid bolus of 40 mL/kg of
normal saline and was placed on maintenance fluids overnight. 1% Staphylococcus aureus

The next morning on the inpatient service, he appears ill with a heart 3 0/0 Haemophilus influenzaetype b
rate of 156 beats per minute, a respiratory rate of 53 breaths per
minute, and an oxygen saturation of 95% while he breathes ambient 86/ Enterovirus
air.
1% Trypanosoma cruzi
Laboratory findings are as follows:
Patient value Reference range 8% Influenza A
Leukocyte
3 count (per 13,000 5500-15,500
mm )
Hemoglobin (g/dL) 11.0 11.5-13.5 KEY LEARNING POINT
C-reactive protein 140 0-5
( mg/liter) The pathogen most likely to cause acute myocarditis
Erythrocyte 30 0-13 in children is an enterovirus.
sedimentation rate
( mm/hour)
u21
An electrocardiogram reveals a prolonged QT interval and diffuse ST-
segment elevation. Additional testing reveals a troponin-1 level of 208 RESOURCES
ng/mL (reference range, <0.1) and a creatine kinase-MB level of 46
ng/mL (<7.7).
CHALLENGE US NEXT QUESTION

Dashboard My Reports My Schedule Help About Contact Us Search Learning technology 1 area9 Sign Out

NEJM Knowledge+ is a product of NEJM Group, a division of the Massachusetts Medical Society. Copyright ©2020 Massachusetts Medical Society. All rights reserved.
myknowledgeplus.nejm.org

NEJM Knowledge+ N Pediatrics Board Review

EJM
Knowledge+ Pediatrics Board Reviews
smc

A previously healthy 9-year-old girl reports that every morning for the
past 3 months, she has become overly tired while running in gym class. YOUR ANSWER IS CORRECT 86%
After barely starting a lap around the running track, she must stop to
catch her breath; she usually feels dizzy and occasionally has chest Which one of the following diagnoses is most likely in this case?
pain. Her symptoms have worsened progressively, and she is
embarrassed to always finish last among her classmates. She feels
better after she rests. She reports no palpitations, syncope,
1% Asthma
musculoskeletal tenderness, fevers, cough, or congestion.

or Pulmonary embolism
On examination, she is afebrile, with a heart rate of 100 beats per
minute, a blood pressure of 94/57 mm Hg, a respiratory rate of 16 49% Dilated cardiomyopathy
breaths per minute, and an oxygen saturation of 99% while breathing
ambient air. She appears thin and is in no apparent distress. Her 14% Partial anomalous pulmonary venous return
cardiac examination is notable for a 2/6 blowing holosystolic murmur,
loudest at the left lower sternal border. There is a single, loud second 36/0 Pulmonary arterial hypertension
heart sound without respiratory variation. There are no retractions, and
both lung fields are clear to auscultation. Her liver edge is 2 cm below
the right costal margin. Her radial and femoral pulses are 2+ without
delay, and her capillary refill time is <2 seconds. She does not have calf KEY LEARNING POINT
tenderness.
The most likely diagnosis in a school-age child with
exercise intolerance and dyspnea on exertion, a
An electrocardiogram (figure 1) and chest radiograph (figure 2) are
single and loud second heart sound, and a
obtained.
noncongested chest radiograph with enlargement of
the main pulmonary artery is pulmonary arterial
hypertension.
1ED
aVR V1 V4 RESOURCES
)
r (

ta:C. V2 VS

CHALLENGE US NEXT QUESTION

Dashboard My Reports My Schedule Help About Contact Us Search Learning technology 1 area9 Sign Out

NEJM Knowledge+ is a product of NEJM Group, a division of the Massachusetts Medical Society. Copyright ©2020 Massachusetts Medical Society. All rights reserved.
myknowledgeplus.nejm.org O rh
NEJM Knowledge+ N Pediatrics Board Review

EJM
Knowledge+ Pediatrics Board Reviews
Sync

A 12-year-old girl presents to the emergency department with pain,


YOUR ANSWER IS CORRECT 86%
redness, and swelling of the left knee for the past day. She has had no
recent fevers, rash, congestion, diarrhea, or neurologic symptoms. She
was diagnosed with a viral pharyngitis 3 weeks ago and received Which one of the following additional treatments, if any, is most appropriate
supportive care. Last week, she had pain and swelling of the right wrist for this patient?
that she thought was related to the start of volleyball season. The pain
resolved after 48 hours of ibuprofen, which the patient then
discontinued. She did not seek medical care at the time. She has no 47% No additional treatment is necessary
family history of autoimmune disease.
0 % Methotrexate

On examination, she has a temperature of 37.4°C, a heart rate of 102


beats per minute, a blood pressure of 100/72 mm Hg, a respiratory 4% Vancomycin and gentamicin
rate of 18 breaths per minute, and an oxygen saturation of 98% while
26/ High-dose aspirin
breathing ambient air. She appears generally tired but nontoxic.
Cardiac examination reveals a 2/6 blowing holosystolic murmur, best 23% Prednisone with taper
heard at the cardiac apex, with radiation to the left axilla. Both lungs
are clear to auscultation. There is no hepatosplenomegaly and no
rashes or skin nodules. The patient has 2+ distal pulses with a capillary
refill time of <2 seconds. Her left knee is slightly warm and tender to
palpation, with moderate swelling and erythema.
In a patient with acute rheumatic fever with
Laboratory findings are as follows: migratory polyarthritis and mild carditis, the most
Patient value Reference range appropriate initial treatment after administration of
appropriate antibiotics is high-dose aspirin.
Leukocyte count (per 7000 4500-13,500
mm 3 )
Platelet count (per 175,000 150,000-450,000 RESOURCES
mm 3 )
Hemoglobin (g/dL) 12.0 11.5-15.5
CHALLENGE US NEXT QUESTION
C-reactive protein 48 0-5

Dashboard My Reports My Schedule Help About Contact Us Search Learning technology 1 area9 Sign Out

NEJM Knowledge+ is a product of NEJM Group, a division of the Massachusetts Medical Society. Copyright ©2020 Massachusetts Medical Society. All rights reserved.
myknowledgeplus.nejm.org O rh
NEJM Knowledge+ N Pediatrics Board Review

EJM
Knowledge+ Pediatrics Board Reviews
Sync

Laboratory findings are as follows: YOUR ANSWER IS CORRECT


Patient value Reference range
Leukocyte count (per 7000 4500-13,500 Which one of the following additional treatments, if any, is most appropriate
mm 3 ) for this patient?
Platelet count (per 175,000 150,000-450,000
mm 3 )
Hemoglobin (g/dL) 12.0 11.5-15.5 47% No additional treatment is necessary

C-reactive protein 48 0-5 0% Methotrexate


( mg/liter)
Erythrocyte 77 0-10 4% Vancomycin and gentamicin
sedimentation rate
( mm/hr) 26% High-dose aspirin

A rapid streptococcal test is negative and serum anti-deoxyribonuclease 23% Prednisone with taper
B antibody testing and antistreptolysin 0 titers are positive. Blood and
synovial-fluid culture results are pending. A joint aspirate reveals clear
fluid with a negative Gram stain and normal erythrocyte and leukocyte
counts.

In a patient with acute rheumatic fever with


A chest radiograph is normal. An electrocardiogram shows sinus
migratory polyarthritis and mild carditis, the most
tachycardia with a normal PR interval and borderline low voltage. An
appropriate initial treatment after administration of
echocardiogram shows normal biventricular structure and function, a
appropriate antibiotics is high-dose aspirin.
structurally normal mitral valve with mild-to-moderate mitral
regurgitation, no aortic regurgitation, and no pericardial effusion.
RESOURCES
A 600,000-unit dose of benzathine penicillin G is administered.

CHALLENGE US NEXT QUESTION


1111..■.=•%mrtl-

Knowledge.

Dashboard My Reports My Schedule Help About Contact Us Search Learning technology 1 area9
NEJM Knowledge+ is a product of NEJM Group, a division of the Massachusetts Medical Society. Copyright ©2020 Massachusetts Medical Society. All rights reserved.
myknowledgeplus.nejm.org

NEJM Knowledge+ N Pediatrics Board Review

EJM
Knowledge+ Pediatrics Board Reviews
smc

CASE YOUR ANSWER IS CORRECT

A 7-day-old male infant is brought to the emergency department by his Which one of the following medications is most appropriate for this patient?
mother for evaluation of poor feeding. He was born full term and had no
complications at birth. All of his mother's prenatal laboratory results
were normal. He is breastfed, but his mother is considering switching
him to a bottle because he appears to tire quickly during feeds and has 3 -/c, Furosemide
not gained weight since hospital discharge. Today his mother tried to
2/ Epinephrine
feed him with a bottle, but he has not been interested in eating since
this afternoon and has had fewer wet diapers than usual.
2/ Ceftriaxone

On examination, he is afebrile and has a heart rate of 170 beats per 5Y, Milrinone
minute, a respiratory rate of 74 breaths per minute, and a blood
pressure of 69/35 mm Hg in the upper right extremity. A pulse 88x, Prostaglandin El
oximeter, placed on his right palm, reads 78% while he breathes
ambient air. His tone is low, and he appears cyanotic. Capillary refill is 4
seconds.

An intravenous line is placed, and a series of laboratory tests are


In a neonate with cyanosis, poor weight gain, and a
ordered.
suspected ductal-dependent cardiac lesion, it is
critical to initiate prostaglandin El.

RESOURCES

CHALLENGE US NEXT QUESTION

Dashboard My Reports My Schedule Help About Contact Us Search Learning technology 1 area9 Sign Out

NEJM Knowledge+ is a product of NEJM Group, a division of the Massachusetts Medical Society. Copyright ©2020 Massachusetts Medical Society. All rights reserved.
myknowledgeplus.nejm.org

NEJM Knowledge+ N Pediatrics Board Review

EJM
Knowledge+ Pediatrics Board Reviews
smc

QUESTION 86%

A 5-year-old girl presents after visiting the emergency department (ED) For which one of the following conditions is this patient at increased risk?
2 weeks ago for an episode of respiratory distress and wheezing
related to a viral illness. A chest radiograph was obtained during the ED
visit (figure), and the patient was treated with supportive care. Her
blood pressure at the time was elevated (132/87 mm Hg).

Today, she is in no acute distress. Her body weight is 16.7 kg, her Hyperaldosteronism
height is 106 cm, and her heart rate is 88 beats per minute. Her blood
pressure is 127/64 mm Hg in the right arm and 112/60 mm Hg in the Polycystic kidney disease
right leg. She is acyanotic and breathing ambient air comfortably. Her
lung fields are clear to auscultation. There are normal first and second Ehlers-Danlos syndrome

1111 11111
heart sounds, with an aortic ejection click, and no murmur or gallop.
Intracranial aneurysm
Her abdomen is soft and nontender; the liver edge is palpable at the
right costal margin. There is no extremity edema or clubbing; she has
Pheochromocytoma
2+ bilateral radial pulses and 1+ left and right femoral pulses.

Other than the viral illness 2 weeks ago, the patient has been healthy.
She has been growing and developing normally. HOW CONFIDENT ARE YOU?

I know it Think so
MI=No idea

Dashboard My Reports My Schedule Help About Contact Us Search Learning technology 1 area9 Sign Out

NEJM Knowledge+ is a product of NEJM Group, a division of the Massachusetts Medical Society. Copyright ©2020 Massachusetts Medical Society. All rights reserved.
myknowledgeplus.nejm.org

NEJM Knowledge+ 'NN Pediatrics Board Review

EJM
Knowledge+ Pediatrics Board Reviews
Sync

Other than the viral illness 2 weeks ago, the patient has been healthy.
She has been growing and developing normally.

For which one of the following conditions is this patient at increased risk?

Hyperaldosteronism

Polycystic kidney disease

Ehlers-Danlos syndrome

Intracranial aneurysm
111.11.11MITITINIME
Pheochromocytoma

HOW CONFIDENT ARE YOU?

I know it Think so
&MN No idea

7
Dashboard My Reports My Schedule Help About Contact Us Search Learning technology 1 area9
NEJM Knowledge+ is a product of NEJM Group, a division of the Massachusetts Medical Society. Copyright ©2020 Massachusetts Medical Society. All rights reserved.
myknowledgeplus.nejm.org 0 1-1) Lr
NEJM Knowledge+ (r Pediatrics Board Review

(REJM
Knowledge+ Pediatrics Board ReviewEl
Sync
M

A 9-year-old girl is brought in by her parents, who report that she has
complained of feeling dizzy several times during the past month while YOUR ANSWER IS INCORRECT 86%
playing with friends on the playground. During these episodes, she has
looked pale and needed to stop and rest for a few minutes. Her
Which one of the following cardiac conduction abnormalities is the cause of
teachers also note that she has seemed more tired than usual during
this patient's symptoms?
the past 4 to 6 weeks. She has no significant medical history and takes
no medications.

On physical examination, she appears well and is alert and interactive. 12% Second-degree atrioventricular block, Mobitz type 2
She has a heart rate of 54 beats per minute, a respiratory rate of 24
breaths per minute, a blood pressure of 104/40 mm Hg, and an oxygen 78% Third-degree complete atrioventricular block
saturation of 98% while breathing ambient air. She has a normal
and a split S2. She has a 2/6 short systolic ejection murmur best heard 3% First-degree atrioventricular block
at the left midsternal border. She has normal peripheral pulses, has no
cyanosis, and is warm and well perfused. She has no hepatomegaly. 1% Sinus bradycardia without heart block

An electrocardiogram is obtained (figure). 7% Second-degree atrioventricular block, Mobitz type 1

Electrocardiographic evidence of atrial activity (P


waves) that occurs independent of ventricular
activity (QRS complexes) characterizes the type of

1

V Vi

heart block known as third-degree atrioventricular


block.
_,-.-i-Av-----A-.
RESOURCES
;3 1.

a T

CHALLENGE US NEXT QUESTION

Dashboard My Reports My Schedule Help About Contact Us Search Learning technology k area 9 Sign Out

NEJM Knowledge+ is a product of NEJM Group, a division of the Massachusetts Medical Society. Copyright ©2020 Massachusetts Medical Society. All rights reserved.
myknowledgeplus.nejm.org

NEJM Knowledge+ 'NN Pediatrics Board Review

EJM
Knowledge+ Pediatrics Board Reviews
Sync

YOUR ANSWER IS CORRECT U


A 16-year-old boy presents for a preparticipation physical examination Which one of the following next steps is most appropriate for this patient?
for high school basketball. He is asymptomatic and has no limitation in
physical activity or abnormal growth or development. His parents are
healthy; his younger brother has bicuspid aortic valve that was
identified on echocardiography during workup of an innocent murmur. 52° Clear for all sports participation without further testing
The patient has no other family history of heart disease, sudden cardiac
2/ Recommend FBN1 gene sequencing
death, or unexplained death.

He has a heart rate of 72 beats per minute, an oxygen saturation of 2/ Obtain a cardiac CT angiogram
98%, and a blood pressure of 112/68 mm Hg in the right upper
extremity and 128/76 mm Hg in the right lower extremity. His height is 1% Recommend genetic testing forACTA2 mutations
183 cm (90th percentile), and his weight is 59 kg (41st percentile) for a
BMI of 23.6.
43/, Obtain an echocardiogram

Physical examination reveals a regular heart rate and rhythm without


murmur, click, or gallop. The patient has 2+ bilateral brachial and
femoral pulses without brachiofemoral delay.

Patients who have a first-degree relative with


isolated bicuspid aortic valve should undergo one-
ti me screening with echocardiography.

1ED
RESOURCES

CHALLENGE US NEXT QUESTION

Dashboard My Reports My Schedule Help About Contact Us Search Learning technology 1 area9 Sign Out

NEJM Knowledge+ is a product of NEJM Group, a division of the Massachusetts Medical Society. Copyright ©2020 Massachusetts Medical Society. All rights reserved.
LEI I myknowledgeplus.nejm.org 0 1-1) of

NEJM Knowledge+ (r Pediatrics Board Review

(REJM
Knowledge+ Pediatrics Board Review El
pc_
M

What is the most likely finding on cardiac catheterization in a 17-year-old boy


with acute ST-segment elevation myocardial infarction who has a negative
toxicology panel, no traditional coronary risk factors, and a history of an
unknown but serious febrile illness with conjunctivitis and rash covering his
entire body and mouth when he was 3 years old?

1% Diffuse coronary artery atherosclerosis

1% Normal coronary arteries

0% Coronary artery vasospasm

3% Coronary artery dissection

95% Coronary artery aneurysm with associated thrombosis

maga KEY LEARNING POINT

In an adolescent who presents with acute


myocardial infarction in the absence of known
coronary risk factors, coronary artery aneurysms
with associated thrombosis may be causing cardiac
ischemia as a long-term sequela of Kawasaki
disease.
col
RESOURCES

41 CHALLENGE US NEXT QUESTION

Dashboard My Reports My Schedule Help About Contact Us Search Learning terd- /logv area 9 Sign Out

NEJM Knowledge+ is a product of NEJM Group, a division of the Massachusetts Medical Society. Copyright ©2020 Massachusetts Medical Society. All rights reserved.
myknowledgeplus.nejm.org

NEJM Knowledge+ N Pediatrics Board Review

EJM
Knowledge+ Pediatrics Board Reviews
smc

A 4-year-old boy with no significant medical history presents with


persistent fever. When you saw him 2 days ago for the fever, he had a V YOUR ANSWER IS CORRECT I 85%
negative rapid strep test, but you prescribed amoxicillin for scarlet fever
because he also had a red tongue. He returns today because the fever
In addition to stopping amoxicillin, which one of the following next steps is
has persisted for 6 days now, and he has a new rash. His mother is
most appropriate for this patient?
concerned that he may be having a reaction to the antibiotic.
On physical examination, he has a temperature of 39.0°C, a heart rate
of 140 beats per minute, a blood pressure of 96/51 mm Hg, and a Order tests for Epstein-Barr virus immunoglobulin M and
respiratory rate of 20 breaths per minute. He is very difficult to examine 1%
i mmunoglobulin G
because he is so irritable. In addition to a maculopapular rash on his
torso, he has the physical findings shown in figures 1, 2, and 3. 0% Initiate intravenous oxacillin

0% Initiate treatment with intravenous glucocorticoids

Initiate treatment with intravenous immunoglobulin G and high-dose


aspirin

4% Obtain an electrocardiogram

KEY LEARNING POINT

A child who meets classic clinical criteria for


Kawasaki disease should be treated with
intravenous immunoglobulin G and high-dose
aspirin.

CHALLENGE US NEXT QUESTION

Dashboard My Reports My Schedule Help About Contact Us Search Learning technology 1 area9 Sign Out

NEJM Knowledge+ is a product of NEJM Group, a division of the Massachusetts Medical Society. Copyright ©2020 Massachusetts Medical Society. All rights reserved.
myknowledgeplus.nejm.org

NEJM Knowledge+ (N Pediatrics Board Review

(REJM
Knowledge+ Pediatrics Board ReviewEl
Sync

YOUR ANSWER IS CORRECT MIN 85%


In addition to stopping amoxicillin, which one of the following next steps is
most appropriate for this patient?

Order tests for Epstein-Barr virus immunoglobulin M and


i mmunoglobulin G

OY Initiate intravenous oxacillin

0% Initiate treatment with intravenous glucocorticoids

Initiate treatment with intravenous immunoglobulin G and high-dose


95%
aspirin

4% Obtain an electrocardiogram

A child who meets classic clinical criteria for


Kawasaki disease should be treated with
intravenous immunoglobulin G and high-dose
aspirin.

CHALLENGE US NEXT QUESTION


Knowledge'
t:■==iMINCECI

Dashboard My Reports My Schedule Help About Contact Us Search Learning technology k area 9
NEJM Knowledge+ is a product of NEJM Group, a division of the Massachusetts Medical Society. Copyright ©2020 Massachusetts Medical Society. All rights reserved.
myknowledgeplus.nejm.org

NEJM Knowledge+ N Pediatrics Board Review

EJM
Knowledge+ Pediatrics Board Reviews
smc

YOUR ANSWER IS CORRECT

A 14-year-old high-school student reports two episodes of syncope in Which one of the following management approaches is most appropriate for
the past 3 months. The first episode occurred when her blood was this patient?
being drawn for routine screening, and the second occurred when she
was dissecting a frog in her biology class. In both episodes, she
experienced tunnel vision and sweating before losing consciousness.
4% Holter monitor
Upon awakening, she was immediately alert and knew what had
happened. Observers reported no seizure activity or loss of bowel or
2% Event recorder
bladder control.
The patient has no family history of pacemaker implantation or sudden 2% Tilt-table study
death. Her vital signs and physical examination are within normal limits.
0% Echocardiogram
An electrocardiogram is obtained (figure).
92/ [Reassurance

The most appropriate next step for a patient with a


classic presentation of vasovagal syncope and a
aVL 1/2 I VS normal electrocardiogram is reassurance.

RESOURCES
ttt aVF '3 V6

CHALLENGE US NEXT QUESTION

Dashboard My Reports My Schedule Help About Contact Us Search Learning technology 1 area9 Sign Out

NEJM Knowledge+ is a product of NEJM Group, a division of the Massachusetts Medical Society. Copyright ©2020 Massachusetts Medical Society. All rights reserved.
myknowledgeplus.nejm.org 0 1-1) Lr
NEJM Knowledge+ (r Pediatrics Board Review

(REJM
Knowledge+ Pediatrics Board Review El
Sync
M

YOUR ANSWER IS CORRECT I 85%


An 8-month-old girl with poor weight gain is hospitalized for initiation of Which one of the following complications is this patient at greatest risk of
nasogastric feeds. She has had difficulty with feeding since birth and developing?
becomes diaphoretic when drinking a bottle.
On examination, she has tachycardia and tachypnea, with intercostal
retractions. She has a loud second heart sound and a holosystolic 2/6 32% Intestinal ischemia
murmur, heard best at the left lower sternal border with a palpable
thrill. The liver edge is 3.5 cm below the costal margin. 54% Recurrent respiratory infections

4% Rickets

2% Gastric ulcers

8% Thrombocytosis

Infants with a large left-to-right cardiac shunt and


pulmonary hypertension are at increased risk for
failure to thrive and recurrent respiratory infections.

RESOURCES

CHALLENGE US NEXT QUESTION

Dashboard My Reports My Schedule Help About Contact Us Search Learning technology k area 9 Sign Out

NEJM Knowledge+ is a product of NEJM Group, a division of the Massachusetts Medical Society. Copyright ©2020 Massachusetts Medical Society. All rights reserved.
myknowledgeplus.nejm.org 0 1-1) Lr
NEJM Knowledge+ 1,(-1\:f Pediatrics Board Review

(REJM
Knowledge+ Pediatrics Board ReviewEl
Sync

YOUR ANSWER IS CORRECT I 87%


A 14-year-old boy is brought to the emergency department with Which one of the following treatments is the best next step in managing this
palpitations that started approximately 15 minutes before arrival. He patient?
has no prior medical problems, takes no prescription medications,
denies illicit drug use, and has no known drug allergies.

The patient is afebrile. His heart rate is 162 beats per minute, his blood Diltiazem
pressure is 110/65 mm Hg, and his oxygen saturation is 98% while he
is breathing ambient air. Cardiorespiratory and neurologic examinations 2Y Direct-current cardioversion
are otherwise unremarkable.
7/ Metoprolol
Vagal maneuvers do not change the cardiac rhythm.

An electrocardiogram is obtained (figure).


4/ Amiodarone

88% Adenosine

0
V4

The most appropriate treatment for a RE DETAILS


hemodynamically stable supraventricular
LI aVL V5 tachycardia that does not respond to vagal
maneuvers is intravenous adenosine.

RESOURCES
V6

CHALLENGE US NEXT QUESTION

Dashboard My Reports My Schedule Help About Contact Us Search Learning technology k area9 Sign Out

NEJM Knowledge+ is a product of NEJM Group, a division of the Massachusetts Medical Society. Copyright ©2020 Massachusetts Medical Society. All rights reserved.
myknowledgeplus.nejm.org 0 1-1) Lr
NEJM Knowledge+ 1,(-1\:f Pediatrics Board Review

(REJM
Knowledge+ Pediatrics Board Review El
Sync

YOUR ANSWER IS CORRECT MIN 87%


A previously healthy 16-year-old girl presents to the emergency What is the most appropriate immediate next step for this patient?
department with a 2-day history of shortness of breath, abdominal pain,
low-grade fevers, and nausea with emesis.

14% Intravenous epinephrine


On examination, she has an oxygen saturation of 95% while breathing
ambient air, a heart rate of 120 beats per minute, a respiratory rate of 58% Intravenous diuretic therapy
30 breaths per minute, a temperature of 37.5°C, and a blood pressure
of 110/74 mm Hg. She appears uncomfortable. Her extremities are 2% Abdominal CT
cool with a delayed capillary refill of 3 to 4 seconds. She is dyspneic
with diffuse crackles on auscultation. She has no murmur but does 23% Intravenous fluid bolus
have an audible third heart sound.
3% Intravenous antibiotics

Notable findings on initial laboratory testing are as follows:


Patient value Reference range
Blood urea nitrogen (mg/dL) 40 5-18
Creatinine (mg/dL) 1.2 0.5-1.0
The mainstay of initial treatment for
Aspartate aminotransferase (U/liter) 100 15-40
decompensated heart failure is immediate
Alanine aminotransferase (U/liter) 90 5-35 administration of intravenous diuretics.

A chest radiograph shows cardiomegaly and increased vascular


markings.
RESOURCES

CHALLENGE US NEXT QUESTION

Dashboard My Reports My Schedule Help About Contact Us Search Learning technology k area 9 Sign Out

NEJM Knowledge+ is a product of NEJM Group, a division of the Massachusetts Medical Society. Copyright ©2020 Massachusetts Medical Society. All rights reserved.
myknowledgeplus.nejm.org

NEJM Knowledge+ N Pediatrics Board Review

EJM
Knowledge+ Pediatrics Board Reviews
smc

YOUR ANSWER IS CORRECT

An 11-year-old boy reports that he was standing talking with friends Which one of the following next steps is most appropriate in evaluating this
when he started to feel dizzy and saw "spots" in his field of vision. He child?
briefly lost consciousness and started to fall down, but he was caught
by a classmate. He did not hit his head and was carefully laid on the
ground. He "woke up" within seconds, after which he felt well and acted
normally. 59% Electrocardiogram

He has not had any recent illnesses, he has no medical history, and 5% Tilt-table testing
there is no family history of early cardiac or sudden death.
2% Electroencephalogram
His vital signs are age-appropriate. His heart rate is regular with no
murmurs, rubs, or gallops; he has normal first and second heart 31% Reassurance
sounds; and he has good pulses and perfusion. No focal neurologic
deficits are evident. The rest of his examination is within normal limits. 2% Echocardiogram

KEY LEARNING POINT

In a child with an episode of uncomplicated syncope


who appears well, has no previous medical
problems, and has an unremarkable physical
examination, the most appropriate next step in
evaluation is an electrocardiogram. 1ED
RESOURCES

CHALLENGE US NEXT QUESTION

Dashboard My Reports My Schedule Help About Contact Us Search Learning technology 1 area9 Sign Out

NEJM Knowledge+ is a product of NEJM Group, a division of the Massachusetts Medical Society. Copyright ©2020 Massachusetts Medical Society. All rights reserved.
myknowledgeplus.nejm.org 0 1-1) Lr
NEJM Knowledge+ 1,(-1\,f Pediatrics Board Review

(REJM
Knowledge+ Pediatrics Board ReviewEl
Sync
M

YOUR ANSWER IS INCORRECT I 89%


A 16-year-old boy reports that he recently lost consciousness and had Which one of the following diagnoses is most likely in this case?
abnormal movements while he was having his blood drawn. He has had
similar episodes before, all during periods of stress. During these
episodes, he feels lightheaded, experiences tunneling of vision, and
has to reach out to steady himself to prevent a fall. Witnesses have 1% Complex partial seizures
observed him flop to the ground, sweat, convulse for a few seconds,
97% Vasovagal syncope
and remain confused for a few seconds afterward.

An MRI of the brain is normal, and there is no epileptiform activity on 0% Hypoglycemia


electroencephalogram.
0% Migraine

1% Juvenile myoclonic epilepsy

KEY LEARNING POINT

The most likely diagnosis in an adolescent with


episodic lightheadedness, pallor, and sweating that
are typically triggered by stress and culminate in a
temporary loss of consciousness is vasovagal
syncope.

RESOURCES

CHALLENGE US NEXT QUESTION

Dashboard My Reports My Schedule Help About Contact Us Search Learning technology k area 9 Sign Out

NEJM Knowledge+ is a product of NEJM Group, a division of the Massachusetts Medical Society. Copyright ©2020 Massachusetts Medical Society. All rights reserved.
myknowledgeplus.nejm.org 0 1-1) Lr
NEJM Knowledge+ (r Pediatrics Board Review

(REJM
Knowledge+ Pediatrics Board Review El
Sync
M

YOUR ANSWER IS CORRECT I 89%


A 16-year-old girl has had three separate blood pressure (BP) measurements
of roughly 130/80 mm Hg. What is the correct interpretation of her BP?

19% Elevated BP (previously described as "prehypertension")

66% Stage 1 hypertension

9% Uninterpretable in the current circumstances

2% Normal BP

4% Stage 2 hypertension

In children a.13 years of age, stage 1 hypertension


is diagnosed on the basis of repeated BP
measurements between 130/80 and 139/89 mm
Hg.

RESOURCES

41 CHALLENGE US NEXT QUESTION

Dashboard My Reports My Schedule Help About Contact Us Search Learning technology k area 9 Sign Out

NEJM Knowledge+ is a product of NEJM Group, a division of the Massachusetts Medical Society. Copyright ©2020 Massachusetts Medical Society. All rights reserved.
myknowledgeplus.nejm.org

NEJM Knowledge+ N Pediatrics Board Review

EJM
Knowledge+ Pediatrics Board Reviews
smc

YOUR ANSWER IS CORRECT

A 14-year-old girl who was born in Mexico and has a visible incisional In addition to referring this patient for further cardiac evaluation, which one of
sternal scar is referred by her dentist for evaluation before urgent the following recommendations is most appropriate with respect to the dental
dental work and cleaning. Her parents report that she was followed in procedure?
the past by a pediatric cardiologist, but they haven't seen that provider
for several years. They are not sure what her diagnosis is, but they
know that she was "blue when she was born" and required two heart
2/, Prescribe clindamycin 600 mg by mouth the night before the procedure
surgeries — one shortly after birth and another a couple of months
later. She currently takes a multivitamin with iron and recently reached
menarche. She has no known drug allergies.
8% Prescribe ampicillin 2 g intravenously 4 hours before the procedure

The patient is currently afebrile. Her blood pressure is 90/50 mm Hg in 75% [Prescribe amoxicillin 2 g by mouth one hour before the procedure
the right arm and 120/80 mm Hg in the left arm. She has a regular
10% Proceed without antibiotic prophylaxis
heart rate at 75 beats per minute and an oxygen saturation of 84%
while breathing ambient air.
5% Prescribe cephalexin 2 g by mouth the morning of the procedure
She has a normal first heart sound; a fixed, split second heart sound; a
right parasternal heave; and a soft, early diastolic decrescendo
murmur, best appreciated over the left lower sternal border. She has no
lower-extremity edema. Her right radial pulse is diminished compared
with her left.
When indicated, the antibiotic of choice for the
prevention of endocarditis in a patient having a
dental procedure is amoxicillin.

CHALLENGE US NEXT QUESTION

Dashboard My Reports My Schedule Help About Contact Us Search Learning technology 1 area9 Sign Out

NEJM Knowledge+ is a product of NEJM Group, a division of the Massachusetts Medical Society. Copyright ©2020 Massachusetts Medical Society. All rights reserved.
myknowledgeplus.nejm.org

NEJM Knowledge+ N Pediatrics Board Review

(REJM
Knowledge+ Pediatrics Board ReviewEl
Sync

YOUR ANSWER IS CORRECT 93%

A 15-year-old girl presents to the emergency department reporting What is the most important immediate management for this patient?
palpitations that began one hour ago, during the last quarter of her
basketball game. She appears to be intermittently falling asleep while
you take her history. She has a heart rate of 211 beats per minute, a
respiratory rate of 20 breaths per minute, and a blood pressure of 20% Administer intravenous adenosine
70/50 mm Hg. An electrocardiogram is obtained (figure).
60% Conduct synchronized cardioversion

20% Instruct the patient to perform vagal maneuvers


•vn VI

0% Administer intravenous amiodarone

0% Administer a fluid bolus

The most important initial treatment for


II
supraventricular tachycardia in a patient who is
hemodynamically unstable (as evidenced by
hypotension, altered mental status, or signs of
shock), is synchronized cardioversion.

RESOURCES

CHALLENGE US NEXT QUESTION

Dashboard My Reports My Schedule Help About Contact Us Search Learning technology k area Sign Out

NEJM Knowledge+ is a product of NEJM Group, a division of the Massachusetts Medical Society. Copyright ©2020 Massachusetts Medical Society. All rights reserved.
myknowledgeplus.nejm.org 0 1-1) Lr
NEJM Knowledge+ (r Pediatrics Board Review

(REJM
Knowledge+ Pediatrics Board Reviews
Sync
M

Which one of the following next steps in management is most appropriate for
a 16-year-old girl with attention deficit-hyperactivity disorder; phenotypic
features of Marfan syndrome (dolichocephaly, pectus malformation, positive
thumb sign, flat feet, tall stature, and myopia); and echocardiographic
evidence of aortic dilation (3.9 cm, z score, 3.22) and mild aortic
regurgitation?

15% Referral for immediate surgical intervention

Discussion of the benefits of stimulant medications for attention deficit


6%
hyperactivity disorder

14% Repeat echocardiography in 2 years

55% [Restriction from competitive and contact sport-

10% Genetic testing to check for possible deletion of the elastin gene

KEY LEARNING POINT

Patients with Marfan syndrome and aortic dilation


with aortic regurgitation are at risk for aortic
dissection.

1u3
41 CHALLENGE US NEXT QUESTION

Dashboard My Reports My Schedule Help About Contact Us Search Learning terd— /logv area 9 Sign Out

NEJM Knowledge+ is a product of NEJM Group, a division of the Massachusetts Medical Society. Copyright ©2020 Massachusetts Medical Society. All rights reserved.
myknowledgeplus.nejm.org 0 1-1) Lr
NEJM Knowledge+ N Pediatrics Board Review

(REJM
Knowledge+ Pediatrics Board ReviewEl
Sync
M

■ 1.•
~ 1r1 ' J
1 I! • • *',

Which one of the following management approaches is appropriate as


prophylaxis to prevent sudden cardiac death in a 17-year-old boy with recent
unexplained syncope and severe septal hypertrophy from hypertrophic
cardiomyopathy?

1% Initiate daily digoxin

20% Refer for septal myectomy

2% Arrange for a 30-day cardiac-event monitor

0% Arrange for a 48-hour Holter monitor

77% Refer for placement of an implantable cardioverter-defibrillator

_MK KEY LEARNING POINT


The most appropriate prophylaxis to prevent sudden
cardiac death in a patient with recent unexplained
syncope and severe septal hypertrophy from
hypertrophic cardiomyopathy is placement of an
i mplantable cardioverter-defibrillator.

RESOURCES

41 CHALLENGE US NEXT QUESTION

Dashboard My Reports My Schedule Help About Contact Us Search Learning technology k area 9 Sign Out

NEJM Knowledge+ is a product of NEJM Group, a division of the Massachusetts Medical Society. Copyright ©2020 Massachusetts Medical Society. All rights reserved.
myknowledgeplus.nejm.org 0 1-1) Lr
NEJM Knowledge+ N Pediatrics Board Review

(REJM
Knowledge+ Pediatrics Board Review El
Sync
M

YOUR ANSWER IS CORRECT

An 11-year-old boy with a history of refractory long-QT syndrome What is the most likely cause of this patient's elevated international
presents for postoperative follow-up one week after placement of an normalized ratio?
i mplantable cardioverter-defibrillator (ICD). He has been taking
propranolol and amiodarone for the past year and had a weight-based
dose of warfarin added to his regimen after placement of the ICD. His
international normalized ratio, checked on the morning of the follow-up 82% Effect of amiodarone on the clearance of warfarin

visit, is 6.0 (target range, 2.0-3.0).


11% Effect of amiodarone on thyroid function

1% Exposure to rat poison in the home

0% Effect of propranolol on the accurate measurement of prothrombin time

Effect of amiodarone on the accurate measurement of the prothrombin


6/
ti me

KEY LEARNING POINT

When a child taking warfarin has a highly elevated


international normalized ratio, clinicians should
review both the dosing of the warfarin and the
overall medication list to evaluate for the possibility
of a drug-drug interaction.

CHALLENGE US NEXT QUESTION

Dashboard My Reports My Schedule Help About Contact Us Search Learning technology k area 9 Sign Out

NEJM Knowledge+ is a product of NEJM Group, a division of the Massachusetts Medical Society. Copyright ©2020 Massachusetts Medical Society. All rights reserved.
myknowledgeplus.nejm.org 0 1-1) Lr
NEJM Knowledge+ 1,(-1\:f Pediatrics Board Review

(REJM
Knowledge+ Pediatrics Board Review El
Sync
M

YOUR ANSWER IS CORRECT

What is the most common cardiac anomaly associated with Turner syndrome?

2% Tetralogy of Fallot

90% Bicuspid aortic valve

0% Hypoplastic left heart syndrome

8% Pulmonary stenosis

0% Dextrocardia

KEY LEARNING POINT

The most common cardiac anomaly associated with


Turner syndrome is bicuspid aortic valve.

RESOURCES

41 CHALLENGE US NEXT QUESTION

Dashboard My Reports My Schedule Help About Contact Us Search Learning technology k area 9 Sign Out

NEJM Knowledge+ is a product of NEJM Group, a division of the Massachusetts Medical Society. Copyright ©2020 Massachusetts Medical Society. All rights reserved.
myknowledgeplus.nejm.org 0 1-1) Lr
NEJM Knowledge+ (r Pediatrics Board Review

(REJM
Knowledge+ Pediatrics Board Review El
Sync
M

YOUR ANSWER IS CORRECT

A healthy 15-year-old boy presents for a sports preparticipation Which one of the following next steps is most appropriate for this patient?
evaluation. His father was recently diagnosed with hypertrophic
cardiomyopathy after a murmur was heard during a routine physical
examination. There is no other known family history of cardiac disease
or unexplained sudden cardiac death. The patient is very active Restriction from competitive sports
physically and denies any personal history of syncope or chest pain with
exercise. Cardiac MRI

His electrocardiogram and echocardiogram findings are normal. Referral for placement of an implantable cardioverter-defibrillator

Genetic testing of the father, followed by targeted genetic testing of the


60%
patient

38% Repeat screening echocardiography every other year

KEY LEARNING POINT

The best next step in evaluation for a patient with a


family history of hypertrophic cardiomyopathy who
has a normal electrocardiogram and a normal
echocardiogram is genetic testing of the affected
family member, followed by targeted genetic testing
of the patient.
RESOURCES

CHALLENGE US NEXT QUESTION

Dashboard My Reports My Schedule Help About Contact Us Search Learning technology k area 9 Sign Out

NEJM Knowledge+ is a product of NEJM Group, a division of the Massachusetts Medical Society. Copyright ©2020 Massachusetts Medical Society. All rights reserved.
myknowledgeplus.nejm.org 0 1-1) Lr
NEJM Knowledge+ 1,(-1\:f Pediatrics Board Review

(REJM
Knowledge+ Pediatrics Board ReviewEl
Sync

I 1.\.• VIL41• w ow

not feel any palpitations or chest pain during the episode.


YOUR ANSWER IS CORRECT
His parents say he has never passed out before but often has to rest
more than his peers during sports. He is adopted, and little is known
What is the most likely cause of this patient's syncopal episode?
about his family history.

He has a heart rate of 46 beats per minute, a respiratory rate of 20


breaths per minute, a blood pressure of 95/60 mm Hg, a temperature
3% Hypertrophic cardiomyopathy
of 37.1°C, and an oxygen saturation of 99% while breathing ambient
air. Orthostatic blood pressures are normal. He is in no acute distress Neurocardiogenic syncope
and appears comfortable. His heart has a regular rate and rhythm
without murmurs, rubs, or gallops. The rest of his physical examination 8/ Wolff-Parkinson-White syndrome
is normal.
Brugada syndrome
An electrocardiogram is obtained (figure).
78% Third-degree atrioventricular heart block

.tvR V va V3r

A syncopal episode associated with exercise, chest


V2 VS V4
pain, or palpitations raises suspicion for a cardiac
cause.

6VF V3 V6 V7

RESOURCES

CHALLENGE US NEXT QUESTION

Dashboard My Reports My Schedule Help About Contact Us Search Learning technology k area9 Sign Out

NEJM Knowledge+ is a product of NEJM Group, a division of the Massachusetts Medical Society. Copyright ©2020 Massachusetts Medical Society. All rights reserved.
NEJM Knowledge+ 1,(-1\:f Pediatrics Board Review

(REJM
Knowledge+ Pediatrics Board ReviewEl
Sync

A 17-year-old boy presents with 3 days of progressive shortness of


breath, fatigue, and chest pain. He says that the pain is worse when he YOUR ANSWER IS CORRECT
li es flat or takes a deep breath. Two weeks ago, he had a cold — with
fevers, myalgias, fatigue, and respiratory congestion — that has since Which one of the following conditions is the most likely underlying cause of
resolved. this patient's symptoms?
On examination, he appears to be anxious and is clutching his chest.
He has an elevated jugular venous pressure, bilateral pulmonary rales,
and a heart rate of 112 beats per minute. He is mildly dyspneic at rest 8% Anomalous left coronary artery from the right coronary sinus
but becomes very short of breath with minimal exertion. An
electrocardiogram and chest radiograph are obtained (figures 1 and 2). 86% Parvovirus B19 infection

1% Inflammation of the costosternal junctions

0% Cocaine ingestion

5% Streptococcus pneumoniae infection

F.

The most likely diagnosis in an adolescent who


presents with acute positional chest pain, exertional
dyspnea, a recent viral illness, diffuse ST-segment
elevation on electrocardiogram, and pulmonary
vascular congestion on chest radiograph is viral
perimyocarditis (most commonly due to parvovirus
B19, human herpesvirus 6, or enteroviruses such RESOURCES

CHALLENGE US NEXT QUESTION

Dashboard My Reports My Schedule Help About Contact Us Search Learning technology area9 Sign Out

NEJM Knowledge+ is a product of NEJM Group, a division of the Massachusetts Medical Society. Copyright ©2020 Massachusetts Medical Society. All rights reserved.
myknowledgeplus.nejm.org 0 1-1) Lr
NEJM Knowledge+ 1,(-1\:f Pediatrics Board Review

(REJM
Knowledge+ Pediatrics Board ReviewEl
Sync
M

YOUR ANSWER IS CORRECT 96%

Which one of the following conditions is the most likely underlying cause of
this patient's symptoms?

Anomalous left coronary artery from the right coronary sinus

86% Parvovirus B19 infection

1% Inflammation of the costosternal junctions

0% Cocaine ingestion

5% Streptococcus pneumoniae infection

The most likely diagnosis in an adolescent who


presents with acute positional chest pain, exertional
dyspnea, a recent viral illness, diffuse ST-segment
elevation on electrocardiogram, and pulmonary
vascular congestion on chest radiograph is viral
perimyocarditis (most commonly due to parvovirus
B19, human herpesvirus 6, or enteroviruses such RESOURCES

CHALLENGE US NEXT QUESTION


—a
<REIM
KnoWledge .

Dashboard My Reports My Schedule Help About Contact Us Search Learning technology k area 9
NEJM Knowledge+ is a product of NEJM Group, a division of the Massachusetts Medical Society. Copyright ©2020 Massachusetts Medical Society. All rights reserved.
myknowledgeplus.nejm.org 0 1-1) Lr
NEJM Knowledge+ (r Pediatrics Board Review

(REJM
Knowledge+ Pediatrics Board ReviewEl
Sync
M

YOUR ANSWER IS CORRECT EMI 96%


A previously healthy 5-year-old boy presents with fever and malaise. He Which one of the following tests is most likely to confirm the correct diagnosis
recently recovered from an upper respiratory infection but last night in this patient?
developed a tactile temperature and abdominal pain. This morning, his
mother noted decreased appetite and a diminished activity level,
prompting her to bring him in for evaluation.
4% Troponin testing

On physical examination, the patient has a temperature of 39.8°C, a 2% Lateral neck radiography
heart rate of 170 beats per minute, a respiratory rate of 40 breaths per
minute, and a blood pressure of 100/65 mm Hg. He appears nontoxic 0% Antinuclear antibody testing
but uncomfortable. When asked to take a deep breath, he complains of
chest pain and repositions himself so that he is sitting up and leaning 93% Electrocardiography
forward. On auscultation, a high-pitched scratching sound is heard
along the left fourth intercostal space without any other murmurs. His 1% CT angiography
lungs are clear to auscultation, and he has no hepatosplenomegaly.

The most useful diagnostic test in a patient with


suspected pericarditis is an electrocardiogram.

RESOURCES

CHALLENGE US NEXT QUESTION

Dashboard My Reports My Schedule Help About Contact Us Search Learning technology area9 Sign Out

NEJM Knowledge+ is a product of NEJM Group, a division of the Massachusetts Medical Society. Copyright ©2020 Massachusetts Medical Society. All rights reserved.
C) myKnowieageps.nejm.org
iu Cf
NEJM Knowledge+ @ Pediatrics Board Review
Pediatrics Board Review
- -
asasla era n 1
VWle ge


A 16-year-old boy who moved to the United States frorr Mexico one
Which one of the following cardiac murmurs is most likely to be found on this
patient's physical examination?

year ago reports a 6-month history of fatigue and increi asing dyspnea
on exertion. He states that he has not experienced fev ers cough, or
weight loss. He recalls that at age 12 he was sick with fever, rash and 14 i Midsystolic ejection murmur over the second right intercostal space
multiple painful joints several weeks after having a sor e throat, but he
en Holosystolic murmur, at the lower-left sternal border, that increases with
never saw a physician for that illness. 12%
inspiration

Systolic and diasto is machine-like murmur over the second left


4%
intercostal space

58% [Holosy stol is murmur at the apex

Harsn systolic crescendo-decrescendo murmur in the second left


12%
intercostal space, radiating to the neck

KEY LEARNING POINT

The cardiac murmur most likely to be heard in an


adolescent with a history of rheumatic fever who
presents with dyspnea is one of mitral regurgitation
(holosystolic murmur at the apex).

CHALLENGE US NEXT QUESTION


myknowledgeplus.nejm.org 0 1-1) Lr
NEJM Knowledge+ (r Pediatrics Board Review

(REJM
Knowledge+ Pediatrics Board ReviewEl
Sync
M

YOUR ANSWER IS CORRECT I 94%


Which one of the following findings is most likely on examination of a 6-month-
old boy with a hemodynamically significant patent ductus arteriosus?

Diminished femoral pulses

Elevated diastolic pressure


r
Bounding peripheral pulses

Cyanotic extremities

Decreased oxygen saturation

KEY LEARNING POINT

The physical examination of an infant with


significant left-to-right cardiac shunting often
reveals bounding peripheral pulses.

RESOURCES

41 CHALLENGE US NEXT QUESTION

Dashboard My Reports My Schedule Help About Contact Us Search Learning technology k area 9 Sign Out

NEJM Knowledge+ is a product of NEJM Group, a division of the Massachusetts Medical Society. Copyright ©2020 Massachusetts Medical Society. All rights reserved.
myknowledgeplus.nejm.org

NEJM Knowledge+ N Pediatrics Board Review

EJM
Knowledge+ Pediatrics Board Reviews
smc

YOUR ANSWER IS CORRECT 94%


A 2-month-old girl is brought to the emergency department with a 3-day
history of increasing irritability and respiratory distress. Her mother
states that she becomes extremely irritable and diaphoretic upon Which one of the following diagnoses is most likely in this case?
breastfeeding but that she has not had any fever, congestion, diarrhea,
or known exposure to sick contacts. Her mother reports that she has a
fussy temperament and has had some irritability with feeds since a 46% Ventricular septa! defect
couple weeks after birth.
14% Viral myocarditis
The patient is afebrile and has a heart rate of 180 beats per minute, a
blood pressure of 84/47 mm Hg, a respiratory rate of 67 breaths per 0% Kawasaki disease
minute, and an oxygen saturation of 93% while breathing ambient air.
She is extremely fussy on examination and slightly diaphoretic. Her 26% Anomalous origin of the left coronary artery from the pulmonary artery
cardiac examination is notable for a 2/6 blowing holosystolic murmur
heard loudest at the apex with radiation to the left axilla and an 14% Tachycardia-induced cardiomyopathy
gallop. She has intercostal and subcostal retractions with faint crackles
heard at both lung bases. Her liver edge is 3 cm below the right costal
margin. She is slightly cool to the touch on her distal extremities, with a
capillary refill of 3 to 4 seconds. KEY LEARNING POINT
Blood cultures and a respiratory pathogen panel are pending. Serum
The most likely diagnosis in an afebrile infant
creatinine and aminotransferase levels are within normal limits. The
presenting with heart failure and
leukocyte count is 9000 per mm3 (reference range, 5000-19,500), the
electrocardiographic signs of myocardial ischemia is
C-reactive protein level is 18 mg/liter (0-5), and the erythrocyte
sedimentation rate is 15 mm/hr (0-10). The brain natriuretic peptide
level is >2500 pg/mL (<100).
anomalous origin of the left coronary artery from the
pulmonary artery. 1ED
A chest radiograph (figure 1) and electrocardiogram (figure 2) are
RESOURCES
obtained.

CHALLENGE US NEXT QUESTION

Dashboard My Reports My Schedule Help About Contact Us Search Learning technology 1 area9 Sign Out

NEJM Knowledge+ is a product of NEJM Group, a division of the Massachusetts Medical Society. Copyright ©2020 Massachusetts Medical Society. All rights reserved.
myknowledgeplus.nejm.org 0 1-1) Lr
NEJM Knowledge+ 1,(-1\:f Pediatrics Board Review

(REJM
Knowledge+ Pediatrics Board Review El
Sync
M

YOUR ANSWER IS CORRECT MN 94%


Which one of the following diagnoses is most likely in this case?

46% Ventricular septa! defect

14% Viral myocarditis

0% Kawasaki disease

26% Anomalous origin of the left coronary artery from the pulmonary artery

14% Tachycardia-induced cardiomyopathy

Click to zoom
KEY LEARNING POINT

The most likely diagnosis in an afebrile infant


presenting with heart failure and
electrocardiographic signs of myocardial ischemia is
anomalous origin of the left coronary artery from the
pulmonary artery.

RESOURCES

CHALLENGE US NEXT QUESTION


-11
<REIM
Knowledge'

1=•=.1

Dashboard My Reports My Schedule Help About Contact Us Search Learning technology k area 9
• --. GC=

NEJM Knowledge+ is a product of NEJM Group, a division of the Massachusetts Medical Society. Copyright ©2020 Massachusetts Medical Society. All rights reserved.
myknowledgeplus.nejm.org

NEJM Knowledge+ N Pediatrics Board Review

EJM
Knowledge+ Pediatrics Board Reviews
smc

YOUR ANSWER IS CORRECT

A 9-year-old boy presents for outpatient follow-up after being seen in In addition to basic laboratory evaluation, which one of the following tests is
the emergency department (ED) 7 days ago for fever and malaise. most likely to reveal the correct diagnosis for this patient?
During that ED visit, he had a leukocyte count of 10,000 per mni
(reference range, 4500-13,500) and an erythrocyte sedimentation
rate of 53 mm/hour (0-13); he was diagnosed with a viral illness and
was discharged to home. Since then, his fever has resolved, but his 6% Abdominal ultrasound

appetite remains poor. He has had two episodes of emesis, and he


2% Abdominal radiograph
continues to have intermittent abdominal pain and decreased energy.
This morning, he awoke complaining that his heart was beating fast.
86A [Echocardiogram
On examination, he appears fatigued with mild volume depletion but is
in no acute distress. He has a heart rate of 145 beats per minute, a 4% Chest radiograph

respiratory rate of 23 breaths per minute, an oxygen saturation of 96%


2% Upper gastrointestinal radiograph series
while he breathes ambient air, and a normal blood pressure. His breath
sounds are clear. He has soft first and second heart sounds, in addition
to a systolic murmur heard best at the upper left sternal border. His
abdomen is slightly distended but soft with normal bowel sounds; no
masses are palpated.

The combination of tachycardia, abdominal pain,


and a new cardiac murmur in a child with a recent
history of fevers should raise concern for postviral
myocarditis.
1ED
RESOURCES

CHALLENGE US NEXT QUESTION

Dashboard My Reports My Schedule Help About Contact Us Search Learning technology 1 area9 Sign Out

NEJM Knowledge+ is a product of NEJM Group, a division of the Massachusetts Medical Society. Copyright ©2020 Massachusetts Medical Society. All rights reserved.
myknowledgeplus.nejm.org

NEJM Knowledge+ N Pediatrics Board Review

EJM
Knowledge+ Pediatrics Board Reviews
smc

What is the most likely diagnosis in a previously healthy 9-year-old girl who has
chronic, progressive exercise intolerance; dyspnea on exertion; a single, loud
second heart sound without respiratory variation; a 2/6 blowing holosystolic
murmur, loudest at the left lower sternal border; a chest radiograph showing
cardiomegaly with enlargement of the main pulmonary artery and
noncongested lung fields; and an electrocardiogram that demonstrates right
axis deviation, right ventricular hypertrophy, and right atrial enlargement?

1% Asthma

66% Pulmonary arterial hypertension

21% Partial anomalous pulmonary venous return

1% Pulmonary embolism

12% Dilated cardiomyopathy

KEY LEARNING POINT .11M


The most likely diagnosis in a school-age child with
exercise intolerance and dyspnea on exertion, a
single and loud second heart sound, and a
noncongested chest radiograph with enlargement of
the main pulmonary artery is pulmonary arterial
hypertension.

41 CHALLENGE US NEXT QUESTION

Dashboard My Reports My Schedule Help About Contact Us Search Learning technology 1 area9 Sign Out

NEJM Knowledge+ is a product of NEJM Group, a division of the Massachusetts Medical Society. Copyright ©2020 Massachusetts Medical Society. All rights reserved.
myknowledgeplus.nejm.org 0 ri\-1 Lr
NEJM Knowledge+ N Pediatrics Board Review

(REJM
Knowledge+ Pediatrics Board Review El
Sync
Th

What is the most appropriate approach to blood pressure (BP) management


and activity clearance for an asymptomatic 18-year-old male athlete who has
had three home BPs averaging 133/83 mm Hg and whose physical
examination is otherwise normal?

Initiate treatment with lisinopril; restrict sports activity until BP is


2%
<130/80 mm Hg

Refer for echocardiography; sports activity can be pursued if there is no


12%
left ventricular hypertrophy

Initiate treatment with hydrochlorothiazide; restrict sports activity until


2%
BP is <130/80 mm Hg

Encourage lifestyle modification with no restriction in sports activity;


81%
repeat BP measurements in 3 months

Encourage lifestyle modification; restrict sports activity until BP is


3%
<130/80 mm Hg

Mg KEY LEARNING POINT Al=


In the setting of a preparticipation evaluation for a
young adult athlete with a new diagnosis of stage 1
hypertension, lifestyle modification without athletic

CHALLENGE US NEXT QUESTION

Dashboard My Reports My Schedule Help About Contact Us Search Learning terd— /logv area 9 Sign Out

NEJM Knowledge+ is a product of NEJM Group, a division of the Massachusetts Medical Society. Copyright ©2020 Massachusetts Medical Society. All rights reserved.
myknowledgeplus.nejm.org

NEJM Knowledge+ N Pediatrics Board Review

EJM
Knowledge+ Pediatrics Board Reviews
smc

YOUR ANSWER IS CORRECT Q 96%

A 2-day-old female infant who was born full term is being evaluated What is the most important next step for this patient?
after a nurse reports that she becomes tachypneic every time she cries.
She has also had difficulty feeding and has not been able to latch
properly.
1% Administer ampicillin and gentamicin
On examination, the infant has a heart rate of 173 beats per minute, a
blood pressure of 79/38 mm Hg, a respiratory rate of 43 breaths per 0% Administer a dose of hydrocortisone
minute, and an oxygen saturation of 87% while she breathes ambient
air. Her glucose level is normal. She appears cyanotic and has 2+ Administer a dextrose bolus
femoral pulses. Her mucous membranes are moist, and capillary refill is
2 seconds. A systolic 2/6 ejection murmur is heard best at the left 98% Initiate prostaglandin infusion
upper sternal border.
1% Administer a normal saline bolus

The most appropriate intervention to prevent clinical


deterioration in a newborn with cyanosis,
tachypnea, and diminished oxygen saturation is
prostaglandin to maintain patency of the ductus
arteriosus. 1ED
RESOURCES

CHALLENGE US NEXT QUESTION

Dashboard My Reports My Schedule Help About Contact Us Search Learning technology 1 area9 Sign Out

NEJM Knowledge+ is a product of NEJM Group, a division of the Massachusetts Medical Society. Copyright ©2020 Massachusetts Medical Society. All rights reserved.
myknowledgeplus.nejm.org 0 1-1) Lr
NEJM Knowledge+ N Pediatrics Board Review

(REJM
Knowledge+ Pediatrics Board ReviewEl
Sync
M

YOUR ANSWER IS CORRECT

A 16-year-old girl who is new to your practice presents for a routine well Which one of the following next steps in management is most appropriate for
examination. She reports that she has a recent diagnosis of attention this patient?
deficit-hyperactivity disorder and that she is interested in trying out for
her school's basketball team.

3% Repeat echocardiography in 2 years


On examination, she is noted to have a prominent forehead and to
wear eyeglasses for myopia. She is tall for her age (185 cm, >99th 21°A Genetic testing to check for possible deletion of the elastin gene
percentile) and weighs 50 kg (50th-75th percentile). She has pectus
carinatum and mild scoliosis. Her lungs sound clear, but cardiac 6% Referral for immediate surgical intervention
examination reveals a 2/6 diastolic murmur. Her joints appear to have
increased laxity, and she is able to adduct her entire thumb beyond the Discussion of the benefits of stimulant medications for attention deficit
3%
ulnar border of the palm. Her feet appear flat. hyperactivity disorder

68% Restriction from competitive and contact sports


An echocardiogram shows a dilated aorta, measuring 3.9 cm at the
sinuses of Valsalva (z score, 3.22), mitral valve prolapse, and mild
aortic regurgitation.

Aim KEY LEARNING POINT


Patients with Marfan syndrome and aortic dilation
with aortic regurgitation are at risk for aortic
dissection.

CHALLENGE US NEXT QUESTION

Dashboard My Reports My Schedule Help About Contact Us Search Learning technology k area 9 Sign Out

NEJM Knowledge+ is a product of NEJM Group, a division of the Massachusetts Medical Society. Copyright ©2020 Massachusetts Medical Society. All rights reserved.
myknowledgeplus.nejm.org 0 1-1) Lr
NEJM Knowledge+ (r Pediatrics Board Review

(REJM
Knowledge+ Pediatrics Board ReviewEl
Sync
M

YOUR ANSWER IS CORRECT I 94%

Which one of the following diagnoses is most likely in a 16-year-old boy with
pleuritic anterior chest pain for 7 days, a recent upper respiratory tract
infection, a friction rub that is audible throughout the respiratory cycle, no
jugular venous distention, normal vital signs, and electrocardiographic
evidence of diffuse T-wave inversions but no ST-segment changes?

0% Acute coronary syndrome

0% Cardiac tamponade

Acute pericarditis

0 /_ Pulmonary embolism

Hypertrophic cardiomyopathy

KEY LEARNING POINT AIM


The most likely physical examination finding in a
patient with pericarditis is a friction rub.

41 CHALLENGE US NEXT QUESTION

Dashboard My Reports My Schedule Help About Contact Us Search Learning technology k area 9 Sign Out

NEJM Knowledge+ is a product of NEJM Group, a division of the Massachusetts Medical Society. Copyright ©2020 Massachusetts Medical Society. All rights reserved.
myknowledgeplus.nejm.org 0 1-1) Lr
NEJM Knowledge+ 1,(-1\:f Pediatrics Board Review

(REJM
Knowledge+ Pediatrics Board Review El
Sync
M

YOUR ANSWER IS CORRECT

A 17-year-old boy presents to the emergency department with sudden- Which one of the following findings is most likely on cardiac catheterization?
onset, crushing substernal chest pain of 5 hours' duration. He has
never had pain like this before. His medical history is notable only for a
hospitalization at age 3 for a febrile illness that lasted one week and
was associated with a whole-body rash (including his mouth) and 90% Coronary artery aneurysm with associated thrombosis

conjunctivitis. His mother recounts him being very ill but does not recall
0% Diffuse coronary artery atherosclerosis
the exact diagnosis. He takes no medications and does not smoke
cigarettes or drink alcohol. He has no family history of coronary artery
3% Coronary artery dissection
disease.

On physical examination, he is diaphoretic and appears uncomfortable. 5% Coronary artery vasospasm


His heart rate is 100 beats per minute, and his blood pressure is
2% Normal coronary arteries
130/80 mm Hg. Cardiovascular examination is notable for tachycardia.
Laboratory evaluation reveals a troponin-I level of 0.6 pg/liter
(reference range, <0.1); his glycated hemoglobin and lipid levels are
within normal range. A toxicology panel is negative. Electrocardiography
KEY LEARNING POINT
shows sinus tachycardia with ST-segment elevations in leads V1-V3.

The patient is taken for emergency cardiac catheterization. In an adolescent who presents with acute
myocardial infarction in the absence of known
coronary risk factors, coronary artery aneurysms
with associated thrombosis may be causing cardiac
ischemia as a long-term sequela of Kawasaki
disease.
RESOURCES

CHALLENGE US NEXT QUESTION

Dashboard My Reports My Schedule Help About Contact Us Search Learning technology area9 Sign Out

NEJM Knowledge+ is a product of NEJM Group, a division of the Massachusetts Medical Society. Copyright ©2020 Massachusetts Medical Society. All rights reserved.
myknowledgeplus.nejm.org

NEJM Knowledge+ N Pediatrics Board Review

EJM
Knowledge+ Pediatrics Board Reviews
smc

YOUR ANSWER IS CORRECT 96%

A 5-year-old boy is brought in by his mother for evaluation of weakness Cardiac evaluation is required in this patient to assess for which one of the
that she first noticed about 6 months ago. It started in his thighs, but following conditions?
his arms have become progressively weaker as well. Recently, she has
noticed that he has been falling and is no longer able to run as well as
his peers. He has not experienced any trauma, been ill, or received any
vaccines recently. He has met all his developmental milestones on time 3% Mitral valve prolapse
and began walking at about 13 months of age.
54% Left ventricular dysfunction
On physical examination, his vital signs are within normal limits for his
age. He is at the 70th percentile for weight and the 60th percentile for Aortic insufficiency
height. His cardiac, respiratory, and abdominal examinations are
normal. Cranial nerves 1I-X11 are intact. His calf muscles appear to be 6% Arrhythmias
enlarged. When asked to rise from a chair, he appears weak and must
37% Hypertrophic cardiomyopathy
walk his hands up his legs to stand upright. He then walks on his
tiptoes.

Laboratory testing reveals a creatine kinase level of 17,000 U/liter


(reference range, 20-200). An electrocardiogram and echocardiogram
are ordered.
In patients with Duchenne muscular dystrophy,
echocardiograms are required at diagnosis and
serially over time to evaluate for left ventricular
dysfunction.
1ED
RESOURCES

CHALLENGE US NEXT QUESTION

Dashboard My Reports My Schedule Help About Contact Us Search Learning technology 1 area9 Sign Out

NEJM Knowledge+ is a product of NEJM Group, a division of the Massachusetts Medical Society. Copyright ©2020 Massachusetts Medical Society. All rights reserved.
myknowledgeplus.nejm.org

NEJM Knowledge+ N Pediatrics Board Review

EJM
Knowledge+ Pediatrics Board Reviews
smc

YOUR ANSWER IS CORRECT

A 10-year-old girl reports sporadic episodes of chest pain localized to What is the most likely diagnosis in this case?
her left chest. The episodes last only a few seconds. The pain is worse
with inspiration, necessitating shallow breaths during the episode. The
pain is not brought on by exertion, and there are no associated
palpitations, dizziness, or exercise intolerance. There is no history of 0/ Mitral-valve prolapse
recent fever or cough.
6/ Costochondritis
The girl was previously healthy, with no hospitalizations. There is no
family history of sudden cardiac death or hypercholesterolemia. Tietze's syndrome

Her vital signs are normal. She is in normal sinus rhythm. She has no 83/ [Precordial catch syndrome
murmur. There are no rubs or gallops. Her radial pulses are 2+. She
has symmetric breath sounds with no wheezes or crackles. Her chest 3% Slipping rib syndrome
wall has no rash, erythema, ecchymoses, or swelling. On palpation of
the left chest, there is no tenderness. The pain is not reproducible by
hooking the lower ribs and pulling them anteriorly. The rest of the
physical examination is normal.

In a child with episodic chest pain, the most likely


cause is noncardiac in origin.

1ED
RESOURCES

CHALLENGE US NEXT QUESTION

Dashboard My Reports My Schedule Help About Contact Us Search Learning technology 1 area9 Sign Out

NEJM Knowledge+ is a product of NEJM Group, a division of the Massachusetts Medical Society. Copyright ©2020 Massachusetts Medical Society. All rights reserved.
myknowledgeplus.nejm.org 0 1-1) Lr
NEJM Knowledge+ (r Pediatrics Board Review

(REJM
Knowledge+ Pediatrics Board Review El
Sync
M

YOUR ANSWER IS CORRECT Li


A 17-year-old boy with no significant medical history is evaluated Which one of the following management approaches is most appropriate for
following an episode of syncope that occurred while he was playing this patient?
basketball and that was preceded only by brief lightheadedness. He
regained consciousness spontaneously after approximately 20
seconds.
4% Arrange for a 48-hour Hotter monitor
Vital signs and physical examination are normal, with the exception of a
2/6 crescendo-decrescendo murmur that was louder with a handgrip 60% Refer for placement of an implantable cardioverter-defibrillator
maneuver and softer with squatting. An echocardiogram reveals normal
left ventricular systolic function, no significant valvular abnormalities, 2% Arrange for a 30-day cardiac-event monitor
and severe septal hypertrophy, with a measured thickness of 3.2 cm.
2% Initiate daily digoxin

32% Refer for septal myectomy

The most appropriate prophylaxis to prevent sudden


cardiac death in a patient with recent unexplained
syncope and severe septal hypertrophy from
hypertrophic cardiomyopathy is placement of an
i mplantable cardioverter-defibrillator.

RESOURCES

CHALLENGE US NEXT QUESTION

Dashboard My Reports My Schedule Help About Contact Us Search Learning technology k area 9 Sign Out

NEJM Knowledge+ is a product of NEJM Group, a division of the Massachusetts Medical Society. Copyright ©2020 Massachusetts Medical Society. All rights reserved.
myknowledgeplus.nejm.org 0 1-1) Lr
NEJM Knowledge+ N Pediatrics Board Review

(REJM
Knowledge+ Pediatrics Board ReviewEl
Sync
M

YOUR ANSWER IS CORRECT

A 16-year-old girl presents for evaluation of primary amenorrhea. She Which one of the following cardiac anomalies is most likely to be found on an
has no significant medical history and takes no medications. She echocardiogram in this patient?
reports that she is not sexually active.

On physical examination, she is 132 cm tall (<1st percentile) and


weighs 49 kg (27th percentile). Her blood pressure is 125/85 mm Hg, 1% Tetralogy of Fallot
and her heart rate is 80 beats per minute. She has a webbed neck
without thyromegaly. She has a broad chest with widely spaced nipples. % Dextrocardia
She is at Tanner stage 1 for breast and pubic hair development.
Cardiac auscultation reveals an early systolic ejection click. 6% Pulmonary stenosis

93% Bicuspid aortic valve

0% Hypoplastic left heart syndrome

The most common cardiac anomaly associated with


Turner syndrome is bicuspid aortic valve.

RESOURCES

CHALLENGE US NEXT QUESTION

Dashboard My Reports My Schedule Help About Contact Us Search Learning technology area9 Sign Out

NEJM Knowledge+ is a product of NEJM Group, a division of the Massachusetts Medical Society. Copyright ©2020 Massachusetts Medical Society. All rights reserved.
myknowledgeplus.nejm.org 0 1-1) Lr
NEJM Knowledge+ (r Pediatrics Board Review

(REJM
Knowledge+ Pediatrics Board Review El
Sync
M

YOUR ANSWER IS CORRECT I 96%


A 16-year-old boy who moved to the United States from Mexico one Which one of the following cardiac murmurs is most likely to be found on this
year ago reports a 6-month history of fatigue and increasing dyspnea patient's physical examination?
on exertion. He states that he has not experienced fevers, cough, or
weight loss. He recalls that at age 12 he was sick with fever, rash, and
multiple painful joints several weeks after having a sore throat, but he
never saw a physician for that illness. 58% Holosystolic murmur at the apex

Harsh systolic crescendo-decrescendo murmur in the second left


12%
intercostal space, radiating to the neck

Holosystolic murmur, at the lower-left sternal border, that increases with


12%
inspiration

14% Midsystolic ejection murmur over the second right intercostal space

Systolic and diastolic machine-like murmur over the second left


intercostal space

KEY LEARNING POINT AMIll


The cardiac murmur most likely to be heard in an
adolescent with a history of rheumatic fever who
presents with dyspnea is one of mitral regurgitation
(holosystolic murmur at the apex).

CHALLENGE US NEXT QUESTION

Dashboard My Reports My Schedule Help About Contact Us Search Learning technology k area 9 Sign Out

NEJM Knowledge+ is a product of NEJM Group, a division of the Massachusetts Medical Society. Copyright ©2020 Massachusetts Medical Society. All rights reserved.

You might also like